Hepatobiliary Flashcards
A 66-year-old Japanese female is referred by her
primary physician for long-standing biliary colic symptoms. She describes 10 to 15 years of intermittent right upper quadrant (RUQ) pain with nausea that typically resolves after 1 to 2 hours.
She went to the emergency room once 6 years ago and had an ultrasound that showed gallstones. Her medical history is significant for hypertension and osteoporosis. Her vitals and exam are unremarkable.
A repeat RUQ ultrasound ordered by her primary care physician now shows a large 3 cm gallstone, As well as a fixed mass in the fundus, 2 cm in diameter, that appears to originate from the gallbladder wall. The immediate surrounding gallbladder wall is thickened to 8 to 11 mm.
A CBC, basic chemistry, and liver function tests are all within normal limits.
Which of the following is a risk factor for gallbladder carcinoma?
A. Hemolytic anemia
B. Biliary dyskinesia
C. Clonorchis sinensis infection
D. Anomalous union of the pancreaticobiliary ductal
system or pancreaticobiliary maljunction (PBM)
E. Auto-immune diseases
D.
A history of gallstones is common, and 65% to
90% of those with biliary carcinoma, have a history
of gallstones. The relation of gallstones to gallbladder
cancer (GBC) is thought to be mediated by chronic
inflammation.
There is a relatively well-defined sequence of flat-epithelial premalignant changes leading to GBC. Chronic inflammation leads to intermediate low-grade dysplastic changes. Dysplastic progression over time leads to carcinoma-in-situ and finally invasive carcinoma.
Anomalous union of the pancreaticobiliary ductal system, where the pancreatic duct and common bile duct merge outside the wall of the duodenum and form a long common channel, is also associated with an increased risk of GBC. This pancreaticobiliary maljunction (PBM)
leads to chronic reflux of pancreatic enzymes.
The progression to GBC is likely mediated through
an epithelial hyperplasia with resultant papillary or
villous epithelial changes progressing to GBC.
Further evidence that this is a distinct pathway from
chronic inflammation is that the gene alterations of
cancers arising in the setting of cholelithiasis differ
from anomalies of the duct system associated cancers.
Adenomas do occur in the gallbladder and can
progress to cancer. However, this likely occurs much
less commonly than the other two pathways, given
a lack of cancer-related molecular changes in most
of these lesions.
Inflammatory bowel disease is also associated with increased risk for gallbladder carcinoma. Clonorchis infection is associated with risk for cholangiocarcinoma, but it has not been linked to carcinoma of the gallbladder.
Chronic Salmonella typhi or paratyphi infection is, however, associated with increased risk for gallbladder cancer.
Hemolytic anemia may be a cause of bilirubin type gallstones but is not a risk factor for gallbladder cancer, neither is biliary dyskinesia or auto-immune diseases.
Which of the following radiographic findings is associated with the highest incidence of gallbladder carcinoma?
A. Pancreaticobiliary maljunction (PBM) without biliary dilatation identified on magnetic resonance cholangiopancreatography (MRCP).
B. Strongly enhancing thick inner layer and a weakly enhancing or non-enhancing outer layer of the gallbladder wall on the portal phase of a multi-detector CT scan (MDCT).
C. Gallbladder polyp 10-20mm on ultrasound.
D. Gallbladder wall calcifications; “porcelain gallbladder”
E. Asymptomatic gallstone greater than 3 cm in size.
B.
A recent retrospective study of findings on MDCT associated with gallbladder cancer found two patterns most associated with finding malignancy at the time of surgery.
A strongly enhancing thick inner layer and a weakly enhancing or non-enhancing outer layer of the gallbladder wall on multi-detector CT scan was shown to have a 52% to 55% incidence for gallbladder cancer.
A single thick layer with heterogenous enhancement on MDCT had an incidence of 35% to 38% for gallbladder cancer.
Pancreaticobiliary malformation (PBM) without bile duct dilatation have an incidence of biliary tract cancer of 37.9%, of which 93.2% of these were gallbladder cancer.
Based on this data, prophylactic cholecystectomy is recommended for these patients.
The incidence rates of malignancy in gallbladder polyps varies widely in published reports but ranges from 9.6% to 40% for polyps 10 to 20 mm.
The wide variance is related to various imaging modalities used and populations studied.
Polyps >10mm, sessile polyps and rapidly growing polyps are all recommendations for gallbladder removal.
The finding of gallbladder calcifications or a porcelain gallbladder was found to be associated with a malignancy in 6% of cases in a recent systematic literature review. This is a far lower number than the historically quoted figure of approximately 25%.
Attempts in the review were made to limit inherent biases in a review of retrospective studies that favor overestimation but this value likely still overestimates the true incidence.
Given the incidence, the decision to perform a prophylactic cholecystectomy should not be absolute and should be weighed against the risks of surgery for the individual patient. The presence of gallstones is associated with an increased risk of gallbladder cancer.
The size and volume of stone burden have been identified as potential risk factors for developing gallbladder cancer.
However, there is no direct evidence of a causal relationship between gallstones and gallbladder cancer. The risk for gallbladder cancer development with a 3 cm or greater stone has been estimated to be a 2% risk over a 20 year period.
Review of all imaging studies shows a gallbladder tumor invades into the muscularis propria. There is no lymphadenopathy or distant metastases seen. What is the most appropriate next step in management?
A. Referral for definitive treatment with chemotherapy and radiation.
B. Referral for neoadjuvant chemotherapy and radiation.
C. Schedule for cholecystectomy.
D. Schedule for cholecystectomy with removal of
regional lymph nodes and en-bloc hepatic resection.
D.
The patient has a T1b lesion or early gallbladder
cancer.
Surgical resection is the only curative therapy
for gallbladder cancer.
The general consensus is for radical cholecystectomy for T1b or greater lesions. T1b tumors have been shown to have lymph node metastasis in 24% of cases. Lower recurrence rates and improved survival have been observed with radical resection including lymph nodes when compared to simple cholecystectomy.
A review of the SEER database showed that the evaluation of even a single lymph node improved overall survival and that radical resection without lymph node assessment was no better than cholecystectomy alone for early stage gallbladder cancer.
Neoadjuvant therapy has been evaluated in the setting of borderline and unresectable extra-hepatic biliary malignancies with good results in survival and obtaining negative margins but none of these addressed gallbladder cancer specifically and were confined to advanced disease.
Adjuvant chemoradiation does have a role in select
gallbladder cancer patients, especially with positive nodes or margins to improve local control.
The same patient with the same history of present
illness and past medical history instead presents
to the emergency room with RUQ pain.
The ultrasound in this case is read as 2 large gallstones
with diffuse gallbladder wall thickening up to 11 mm, pericholecystic fluid, and a normal common bile duct. Labs show white blood cell count of 13 and normal liver function. You take the patient for laparoscopic cholecystectomy.
There was no concern for malignancy during the
procedure. The gallbladder was removed without
spillage and with a retrieval bag. You see her 2 weeks
later in clinic, and review of the pathology report
shows T2 adenocarcinoma.
All surgical margins, including cystic duct margin, are reported as clear. What is the most appropriate course of action?
A. No additional surgery, surveillance with imaging
every 6 months.
B. Staging with imaging followed by radical cholecystectomy to include: liver resection with at
least 3 cm of margin around gallbladder bed and regional lymphadenectomy.
C. Staging with imaging followed by radical cholecystectomy and in addition, excision of the
previous laparoscopic port sites.
D. Staging with imaging followed by radical cholecystectomy and in addition, excision of the
common bile duct.
E. Staging with imaging followed by radical cholecystectomy and in addition, excision of both the
common bile duct and the laparoscopic port sites.
B.
Prognosis of gallbladder carcinoma is determined
by the depth of tumor infiltration and the ability to
obtain a tumor-free resection margin (R0).
For T2 or greater the definitive resection should include a minimal hepatic resection centered on the gallbladder
bed and a regional lymphadenectomy.
The majority of experts also extend this recommendation to T1b tumors but some controversy persists. There is uniform agreement that cholecystectomy alone is sufficient for Tis and T1a tumors. There is also general
agreement regarding several other technical points, such as resection of the common bile duct, and the need for port site excision. Excision of the common bile duct is only necessary for a positive cystic duct margin or direct invasion of the hepatoduodenal ligament.
Regarding routine common bile duct excision with radical cholecystectomy unless there is direct invasion of the hepatoduodenal ligament and/or of the cystic duct, bile duct resection does not result in decreased recurrence or better overall survival and does not increase the number of nodes in the specimen.
Peritoneal involvement with gallbladder cancer is common and there is theoretical adverse impact on this with pneumoperitoneum. However, the risk of port site recurrence is based on perforation of the gallbladder or extraction without a retrieval bag rather than the pneumoperitoneum.
Port-site excision does not improve overall or disease-free survival in large retrospective series. Port site excision does not need be routinely performed during secondary procedures for gallbladder cancer discovered
after laparoscopic cholecystectomy.
Which of the following is true regarding the surgical management for T2 or T3 gallbladder cancers?
A. Formal segmentectomy (4b + 5) improves overall survival over wedge resection.
B. Formal segmentectomy reduces local recurrence
rates compared to wedge resection, but overall survival is the same.
C. Formal segementetomy improves disease free survival, but not overall survival.
D. A clear survival benefit for formal segmentectomy
over wedge resection has not been demonstrated.
D.
Although some studies have reported anatomical resection improves survival and RO resection rate when compared to wedge resection, other reports have not demonstrated a benefit.
The majority of these studies dealt with liver resection for liver metastases. There are a few studies looking at gallbladder cancer specifically.
Pawlik et al. found that patients who underwent a major hepatic resection (e.g., formal segmentectomy of 4b + 5 or hemi-hepatectomy) had a similar risk of disease-specific death compared with patients who underwent a hepatic wedge resection. Horiguchi et al. found the overall survival rate and disease-free survival rate at 5 years did not differ significantly between wedge resection and 4a + 5 resection group for T2 tumors.
The available evidence in gallbladder cancer does not show a clear benefit to anatomic resection.
As such, the surgeon’s goal should be to resect all disease with negative histologic margins and chose the appropriate operation to achieve this with the fewest complications.
A 67-year-old female presents with sharp, burning right upper quadrant abdominal pain that radiates to her back. The pain awakens her from sleep. She has experienced nausea and three episodes of emesis. She reports burning substernal and epigastric pain. She denies any fevers or chills. She is hemodynamically normal and afebrile. Laboratory workup reveals a normal hepatic function panel, a normal basic chemistry, and a normal white blood cell count. Her abdominal exam is remarkable for mild tenderness in the right upper quadrant without peritoneal signs. After three hours, her pain resolves.
- What is the next best test in this patient scenario?
A. Plain films of the abdomen
B. Cholecystokinin stimulated cholescintigraphy
C. Right upper quadrant ultrasound
D. Computed tomography (CT) of the abdomen
E. Esophagogastroduodenoscopy (EGD)
- The appropriate imaging is ordered and shows no abnormality. Which is the next most appropriate
step in management?
A. Cholecystectomy
B. Esophagogastroduodenoscopy
C. Right upper quadrant ultrasound in two weeks
D. Proton pump inhibitor administration
E. Bile microscopy
1) C. This scenario describes symptomatic cholelithiasis, or biliary colic.
Plain films of the abdomen would not be a high yield study as less than 10% of gallstones are radio-opaque, nor would it offer specific imaging of the biliary system. Other yields from a plain film would be presence of nephrolithiasis or free air (Choice A).
A hepatobiliary (HIDA) scan is the gold standard for the diagnosis of acute cholecystitis—a clinical scenario in which she does not fit criteria (Choice B). With the cholecystokinin injection, the nuclear imaging study can be diagnostic for biliary dyskinesia.
CT can often visualize gallstones but is not the recommended imaging modality of choice for cholelithiasis as the sensitivity is 55% to 80% (Choice D); cholelithiasis is often found incidentally on CT scan when the imaging is performed for other reasons.
Esophagogastroduodenoscopy could be helpful to rule out peptic ulcer disease that can sometimes mimic symptoms of biliary colic, but this test would not be the ideal next choice in this situation (Choice E).
The most common initial imaging modality in biliary disease is the right upper quadrant ultrasound. Ultrasound has a sensitivity of 84% and specificity approaching 99% for the diagnosis of cholelithiasis (Choice C). Simultaneously, assessment for signs of acute cholecystitis, choledocholithiasis, and hepatic pathology can be sought.
2) C.
This patient should have had a right upper quadrant ultrasound performed in the first question above. If this imaging modality shows no abnormality, but the patient has biliary symptoms, she should have a repeat right upper quadrant ultrasound performed in two weeks with focus on areas that may have missed stones smaller than 3 mm (Choice C).
Esophagogastroduodenoscopy is an appropriate diagnostic modality to rule out peptic ulcer disease and can be utilized if the repeat ultrasound is still negative. Bile microscopy has been shown as an adjunct study to assess for microlithiasis and can be helpful (Choice E), although is not routinely performed; bile microscopy also requires endoscopy for collection. The sensitivity of bile microscopy for microlithiasis is 65% to 90%.
Cholecystectomy is not recommended at this time as other causes of the patient’s symptoms have not been ruled out. Cholecystokinin stimulated cholescintigraphy can help identify a functional gallbladder problem that
may warrant cholecystectomy if positive (Choice A).
Proton pump inhibitor therapy may help with reflux
symptoms but does not specifically address biliary
symptoms (Choice D).
A 67-year-old female presents with sharp, burning right upper quadrant abdominal pain that radiates to her back. The pain awakens her from sleep. She has experienced nausea and three episodes of emesis. She reports burning substernal and epigastric pain. She denies any
fevers or chills. She is hemodynamically normal and afebrile. Laboratory workup reveals a normal hepatic function panel, a normal basic chemistry, and a normal white blood cell count. Her abdominal exam is remarkable for mild tenderness in the right upper quadrant without peritoneal signs. After three hours, her pain resolves.
The patient returns to the emergency department one month later with similar symptoms. A right upper quadrant ultrasound is performed that shows gallstones without signs of acute cholecystitis. Her pain again resolves and she is referred to your clinic the following week. You discuss performing a cholecystectomy with the patient, which she refuses. Which of the following
is true regarding medical therapy of cholelithiasis?
A. Ursodiol therapy typically resolves biliary symptoms within one month.
B. Complete resolution of cholelithiasis is successful in less than 40% of patients.
C. If biliary symptoms persist while on ursodiol, cholecystectomy is indicated.
D. The mechanism of action of ursodiol includes reduction in bile synthesis.
E. Extracorporeal shockwave lithotripsy (ESWL)
has a symptomatic recurrence rate of 30%.
B.
Ursodiol may take up to 3 months to show
improvements in biliary symptoms, and can takeup to
3 years to completely dissolve gallstones (Choice A).
A meta-analysis of treatment with ursodiol showed
that only 37% of patients had complete resolution
of biliary symptoms; cholecystectomy remains the
preferred choice in surgical candidates (Choice B).
Many patients who undergo initial medical therapy
with ursodiol do progress to surgical treatment, but
they tend to show initial improvement in symptoms.
If biliary symptoms progress while being treated
with ursodiol, other causes must be ruled out, such
as sphincter of Oddi dysfunction or peptic ulcer
disease (Choice C).
The mechanism of action of ursodiol involves reducing absorption in the duodenum, resulting in disruption of micelles and reduced cholesterol absorption, thereby decreasing cholesterol concentration; ursodiol is not involved with the synthesis of bile (Choice D).
Extracorporeal shockwave lithotripsy can be utilized as nonsurgical therapy, but has a recurrence rate of approximately 20%.
Indications include patients with single stones, between the sizes of 5 mm and 2 cm.
ESWL is not commonly offered as medical therapy due to the efficacy and commonplace practice of cholecystectomy (Choice E).
A 67-year-old female presents with sharp, burning right upper quadrant abdominal pain that radiates to her back. The pain awakens her from sleep. She has experienced nausea and three episodes of emesis. She reports burning substernal and epigastric pain. She denies any fevers or chills. She is hemodynamically normal and afebrile. Laboratory workup reveals a normal hepatic function panel, a normal basic chemistry, and a normal white blood cell count. Her abdominal exam is remarkable for mild tenderness in the right upper quadrant without peritoneal signs. After three hours, her pain resolves.
The patient’s ultrasound, in addition to cholelithiasis, is also notable for a 1.5 cm non-mobile polypoid lesion in the fundus of the gallbladder. Which of the following is true?
A. Polyps greater than 5 mm are a risk factor for
gallbladder cancer.
B. Patients with large (> 2.5 cm) gallstones are more likely to develop gallbladder cancer than those without.
C. Laparoscopic cholecystectomy should be performed, a frozen analysis should be done, and if
positive, an oncologic resection should be done
laparoscopically.
D. Multiple pedunculated subcentimeter lesions are also a risk factor for gallbladder adenocarcinoma.
E. Extended cholecystectomy is not required for gallbladder adenocarcinomas with up to T3 lesions.
B.
This patient should undergo open cholecystectomy with a symptomatic, visualized preoperative intraluminal gallbladder lesion that is 1.5 cm in size.
The open cholecystectomy is preferred, because
gallbladder perforation or bile spillage during laparoscopic cholecystectomy can potentially seed the peritoneal cavity.
Polypoid lesion size greater than 1 cm is an independent risk factor for gallbladder adenocarcinoma, along with stone size greater than 2.5 cm (Choice A, B).
Gallbladder cancer has a 3:1 ratio of incidence in women and typically presents after age 60.
Extended cholecystectomy is not indicated if the lesion is confined below the muscle layer of the gallbladder.
If the lesion is T2 or greater, resection usually includes segments IVb and V of the liver (Choice E).
Choice D is descriptive of cholesterol polyps, which are not a risk factor for gallbladder adenocarcinoma.
A 70-year-old male presents to the emergency department with altered mental status. Family reports he was complaining of right upper quadrant abdominal pain prior to becoming altered mentally. On arrival, he is found to have a temperature of 102.5°F, heart rate of 112, and systolic blood pressure of 80 despite 2 liters of crystalloid infusion. On exam, he is visibly jaundiced with tenderness in the right upper quadrant. He has 3 out of 4 systemic inflammatory response syndrome
(SIRS) criteria. He is started on pipericillin/tazobactam and admitted to the intensive care unit for invasive monitoring and vasopressor support.
- Regarding this patients constellation of symptoms, what is the most common cause?
A. Gallstones
B. Biliary stricture
C. Malignancy
D. Genetic disorder
A.
Cholangitis is caused by obstruction of the biliary tree eventually leading to bile stasis and bacterial infection. The most common cause being gallstones, which account for around half of cases.
Other causes include stenosis/biliary stricture, malignancy and biliary stents.
Stents can cause obstruction from migration, occlusion, or colonization by bacteria leading to bacterial overgrowth and translocation into the bloodstream.
Regarding the pathophysiology of cholangitis, which of the following is correct?
A. Increased biliary pressure leads to decrease in production of IgG in the biliary mucosa leading to increased translocation of duodenal bacteria.
B. Intra-portal toxins and bacteria can cross through the biliary system due to stasis leading to infection.
C. Stones do not colonize with bacteria.
D. Biliary stents are not felt to contribute to or cause
cholangitis as they help decompress the biliary tree.
B.
Obstruction of the biliary tree via stricture/stenosis, stones, malignancy, or stent occlusion leads to increased biliary tract pressure. This pressure promotes stasis of bile and decreases production of IgA in the bile tract mucosa.
The lack of continuous bile flow, coupled with decrease mucosal protection allows for bacterial translocation from the duodenum through the biliary tract. This static bile and gallstones provide a healthy growth medium for bacteria.
The elevated intra-biliary pressure allows for translocation of these pathogens into the systemic
circulation causing septicemia. Less commonly, bacteria and toxins can enter through the portal circulation into the bile due to increase biliary pressure.
The most common bacteria are gram negative enteric pathogens: E coli, klebsiella, and enterobacter. They carry LPS that promotes cytokine release and leads to septic shock. Enterococcus is seen in a smaller set of cases.
Regarding the proper diagnostic workup, which noninvasive test has highest sensitivity and specificity for detecting the most common cause?
A. Abdominal ultrasound
B. Magnetic resonance cholangiopancreatography (MRCP)
C. CT Scan
D. Hepatobiliary (HIDA) scan
B.
MRCP is the best non-invasive test to confirm the presence of choledocholithiasis due to its high sensitivity (some studies quoting 100%) and nearly 100% specificity.
If the test is positive then this confirms diagnosis. It also is helpful in evaluating for stricture and ampullary masses.
Ultrasound is a good screening tool and can evaluate for common bile duct dilatation to perhaps lend clinical suspicion to presence of biliary obstruction. It is best at identifying the presence of cholelithiasis/cholecystitis. However, it has lower accuracy in identifying the presence of a choledocholith, roughly 80%.
A CT scan is less useful than ultrasound in detecting cholecystitis/common bile duct dilatation, but is helpful at evaluating for ampullary masses as a cause of a dilated common bile duct.
HIDA scans are not useful in the setting of cholangitis as the biliary tract infection reduces secretion of the radio nucleotide labeled marker into the biliary tree. It may show, however, obstruction with lack of flow into the duodenum.
Regarding the management of septic cholangitis caused by choledocholithiasis, which of the following is correct?
A. Urgent biliary tract decompression via endoscopic retrograde cholangiopancreatography (ERCP) is successful 60% of the time.
B. Percutaneous transhepatic cholangiography (PTC) is feasible for stone extraction and stent placement.
C. ERCP with a sphincterotomy is equivalent to cholecystectomy for reducing recurrence rates.
D. Should ERCP and PTC fail or are not feasible, operative choledochotomy and T-tube placement should be avoided because of the risk of surgery.
E. Broad spectrum antibiotic therapy alone will generally provide adequate treatment.
B.
In cases of cholangitis without septic shock, a trial of antibiotic therapy is recommended as this can resolve symptoms and ensure stability in up to 80% of patients.
Routine ERCP can be performed in this setting assuming the patient remains stable.
This patient displays Reynold’s pentad of fever, right upper quadrant pain jaundice, altered mental status, and hypotension, the first three signs constituting Charcot’s triad. This lends suspicion to suppurative cholangitis due to the patient’s state of septic shock.
Antibiotics, though required as initial therapy, are unlikely to complete resolve this patient’s septic physiology. Emergent/urgent biliary tree decompression is warranted and must be performed to prevent excessive morbidity/mortality.
ERCP with sphincterotomy has shown upwards of a 95% success rate in stone extraction and decreasing the rate of recurrence of cholangitis. It is, however, not superior to cholecystectomy in decreasing rates of recurrence and thus cholecystectomy is recommended after the ERCP/sphinceterotomy once the septic physiology has resolved. Arguments for early cholecystectomy have been made as waiting 6 to 8 weeks runs the risk of 20% recurrence rate of a gallstone related event.
Should ERCP fail, PTC is warranted as both these
procedures decrease the morbidity/mortality risk of a common bile duct exploration. PTC can be challenging if there is little intra-hepatic ductal dilatation and it also does not allow sphincterotomy.
Common bile duct exploration is warranted should ERCP and PTC fail at decompressing the biliary tree. In the setting of a patient who is in septic shock, choledochotomy with stone extraction and T-tube placement is recommended as this allows for decompression of the biliary tree and allows for sepsis to resolve prior to performing cholecystectomy to limit the morbidity and mortality associated with both procedures.
The exploration is performed through a choledochotomy on the common bile duct distal to the insertion of the cystic duct. Stay sutures are placed on either side of the choledochotomy and using balloon catheters, fluoroscopy with basket retractors, and flushing, stone extraction is performed. In general, the choledochotomy should be roughly the size of the largest stone. It is best done in dilated ducts as the risk of stenosis is high in
the setting of common bile duct size <6 mm.
A large bore T-tube is placed and the choledochotomy repaired over the T-tube with 4-0 absorbable sutures.
The tube is externalized and bile allowed to drain into an external bag. Due to lack of re-absorption of bile, a patient with this procedure is prone to being deficient in fat soluble vitamins (A, D, E, and K). It is the vitamin K deficiency which is most worrisome as it can lead to a coagulopathy.
A 32-year-old gravida 2 para 1 female at 28 weeks gestation presents with acute onset of right upper quadrant and right upper flank pain with associated nausea and vomiting over the preceding 24 hours. She has no significant medical or surgical history. She has had 1 uncomplicated vaginal delivery. At the time of her evaluation, her temperature is 99.8°F, heart rate is 110, and respiratory rate is 24. Her exam documents a positive Murphy’s sign and guarding in the right upper quadrant. Laboratory studies show the following: WBC - 20,000, H/H- 9/29, Platelets 130,000, ALT-60, and AST 90. Her bilirubin, lipase, and amylase levels are normal. Her urinalysis is within normal limits.
- Which of the following diagnoses is the least common disease in pregnancy presenting with right upper quadrant pain?
A. Acute fatty liver of pregnancy
B. Cholecystitis
C. Cholelithiasis
D. HELLP syndrome
E. Appendicitis
A.
The most common surgical disease in pregnancy is appendicitis with an incidence of 1 in 1000 to 2000 pregnancies.
Gallbladder disease is the second most common surgical disease in pregnancy with an incidence of 1 in 1200 to 1 in 10,000. Theoretically, the incidence of gall bladder disease including cholelithiasis, cholecystitis, and cholangitis should be increased in pregnancy.
The elevated level of serum estrogen seen in pregnancy increases cholesterol secretion, whereas the elevated level of progesterone reduces soluble bile acid secretion and slows emptying of gallbladder. Despite the predilection toward biliary sludge and stone formation, cholecystitis does not occur more frequently during pregnancy. Appendicitis occurs with equal frequency in each trimester and the incidence is not increased in the gravid patient.
The differential diagnosis for RUQ abdominal pain is expanded in pregnancy. It includes gastrointestinal disorders such as pancreatitis, peptic ulcer disease, hepatitis, and appendicitis, due to a superiorly displaced cecum, as well as pyelonephritis, nephrolithiasis, right lower lobe pneumonia, peptic ulcer disease, and myocardial infarction.
Obstetric specific diagnoses must also be included in the differential to include preeclampsia, HELLP (Hemolysis, Elevated Liver enzymes. Low Platelets syndrome and acute fatty liver of pregnancy (AFLP).
HELLP syndrome is a severe form of preeclampsia occurring in up to 8 of 1000 pregnancies presenting most commonly in the third trimester of pregnancy. This syndrome generally involves the characteristic hypertension and proteinuria seen with preeclampsia with evidence of liver dysfunction and a consumptive coagulopathy which can rapidly progress to fulminant
DIC.
Patients with preeclampsia may present with right upper quadrant or epigastric pain due to liver involvement and in the most severe cases subcapsular hemorrhage or hepatic rupture. AFLP is a rare diagnosis, seen in 1:20,000 pregnancies.
This patient presents with findings consistent with an inflammatory intra-abdominal process. Cholecystitis, choledocholithiasis, and cholangitis lead the differential diagnosis. The physical exam findings are highly suggestive of gallbladder disease. WBC counts and alkaline phosphatase levels are routinely elevated during pregnancy and therefore may not be as specific for inflammation during the assessment of the gravid patient.
Which of the following statements is correct concerning Acute Fatty Liver of Pregnancy (AFLP)?
A. AFLP presents most commonly in the second trimester of pregnancy.
B. AFLP commonly presents with serum aminotransferase levels similar to those found in gallbladder disease.
C. AFLP can present with hypoglycemia and occasionally renal failure which can help distinguish
it from HELLP (hemoconcentration, elevated liver enzymes, low platelet) syndrome and gallbladder disease.
D. In a preterm pregnancy, it is considered safe to
continue the pregnancy in a patient who has been diagnosed with AFLP.
C.
The AFLP syndrome almost always presents in the third trimester with serum aminotransferase elevations up to 1000IU/L, which is generally higher than those found in gallbladder disease.
AFLP can also present with hypoglycemia and renal failure, which is not characteristic of either HELLP or gallbladder disease.
Findings of AFLP can still significantly overlap with those of HELLP, making it very difficult to distinguish these two syndromes.
The treatment for both is the emergent delivery of the fetus.
Which of the following statements regarding radiographic imaging of biliary disease in pregnancy is
correct?
A. Classic sonographic signs of biliary disease are
altered in pregnancy.
B. The risk of radiation exposure to the fetus with ERCP (endoscopic retrograde cholangiopancreatography) is high throughout pregnancy.
C. The neuronal development of the fetus is most
sensitive to radiation between 20 to 28 weeks gestation.
D. Exposure to less than 5 rad of ionizing radiation has not been associated with an increased risk of fetal anomalies or pregnancy loss.
E. MR imaging has a higher sensitivity and specificity in the diagnosis of cholecystitis than ultrasonography.
D.
Due to the acuity of presentation and the myriad of diagnoses in the differential, imaging is an essential component in the diagnostic evaluation.
Risks of radiologic studies to the fetus must therefore be considered.
Sonography is the appropriate first line diagnostic modality in pregnancy for both biliary disease and appendicitis as this modality has a high diagnostic accuracy (90% to 100% for both diagnoses) and has no known risk to the fetus.
Classic ultrasound (US) findings to include wall edema, pericholecystic fluid, calculi, and sonographic Murphy’s sign maintain their sensitivity and specificity in pregnancy.
If ultrasound studies arc non-diagnostic, MR imaging without contrast has become the confirmatory test for appendicitis.
For biliary disease, an MRCP can be used in equivocal eases or in suspected eases of choledocholithiasis or cholangitis. It is not as sensitive as US for cholecystitis.
Intraoperative cholangiogram in combination with cholecystectomy is an option for diagnostic evaluation after fetal organogenesis is complete in the second trimester and does not appear to increase the risk for preterm delivery or adverse fetal outcomes.
If MRCP documents stone disease in the biliary tree, ERCP is considered a viable therapeutic option after the first trimester. The risks to the fetus with cholangiogram and ERCP can be reduced with shielding.
CT scan of the abdomen, which is the preferred imaging modality for appendicitis in the non-pregnant patient, confers radiation levels of 5 to 10 rads which approach the maximum permissible radiation dose for fetal exposure during pregnancy.
Fetal exposure to ionizing radiation increases risks of microcephaly, micropthalmia, mental retardation, growth restriction, and cataracts.
The concern of ionizing radiation is greatest during organogenesis which falls between 3-20 weeks of gestation.
The above patient is at 28 weeks and therefore the
risk of serious complications with ionizing radiation is limited. CT generally remains behind US and MR on the imaging algorithm for both appendicitis and cholecystitis even in the patient with a fetus of advanced gestational age due to the disputed twofold
increased risk of carcinogenesis ( 1:1000) in the fetus.
CT imaging should, however, not be abandoned as a diagnostic modality, as the risk of delay in diagnosis far outweighs the risk of radiation.
The consulting radiologist can design CT protocols to minimize the associated risks and counseling can minimize the associated anxiety of the patient.
In this patient, acute cholecystitis is diagnosed by ultrasound. Which of the following is correct regarding treatment of this patient?
A. The risk of adverse effects of laparoscopy is high
even with maximal intra-abdominal pressures limited to 9 mm Hg.
B. If left untreated, the most common complication
of acute cholecystitis in pregnancy is gangrenous
cholecystitis.
C. Available studies have shown significant differences regarding preterm delivery rates, birthweights or neonatal outcomes when comparing laparoscopic versus open cholecystectomies.
D. Nonsteroidal anti-inflammatory drugs (NSAID)
treatment for pain expected to last more than
48 to 72 hours is the pharmacologic option of
choice after 30 weeks gestation to avoid fetal
complications.
E. Beta-lactam antibiotics such as ampicillin-sulbactam or piperacillin-tazobactam are contraindicated in pregnant patients.
B.
Initial non-surgical management can be considered in hemodynamically normal pregnant patients experiencing cholelithiasis. This management plan generally involves bowel rest, intravenous hydration, and NSAID therapy.
A short course (< 48 to 72 h) of indomethacin treatment can provide effective analgesia but is generally avoided in late pregnancy due to the potential adverse fetal effects. Use in the third trimester increases the risk of premature closure of the patent ductus arteriosis and oligohydramnios.
Intravenous antibiotics to include ampicillin-sulbactam, piperacillin-tazobactam, and ticarcillin-clavulanate are not contraindicated in pregnant patients who need antibiotics for acute cholecystitis or choledocolithiasis.
Early surgery has been advocated for all types of biliary disease in pregnancy. If not treated, cholecystitis can lead to life threatening complications, the most common of which is gangrenous cholecystitis followed by abscess formation, perforation, fistula, ileus, or emphysematous cholecystitis. For symptomatic cholelithiasis with no evidence of cholecystitis, surgery can be delayed.
However the literature reports that surgical management of symptomatic cholelithiasis in pregnancy is safe, decreases hospital days, reduces emergency room visits, and the rate of preterm deliveries.
The second trimester (from 13 to 27 weeks gestation) is considered the best timeframe for cholecystectomy as the uterus is not obstructing the view of the surgical field, and the risk of miscarriage or preterm birth is lowest.
In this patient, with clear evidence of cholecystitis, surgical intervention is warranted to reduce risk of serious complications. If complications such as cholangitis or gallstone pancreatitis develop in a pregnant patient, maternal mortality may approach 15% and fetal loss up to 60%.
Surgical management of biliary disease has been revolutionized with the advent of laparoscopy. Laparoscopic technique can be utilized safely in pregnancy across all trimesters depending on the comfort level of the surgeon.
Although data is limited, laparoscopy does not confer an increased risk of adverse pregnancy outcomes to include preterm delivery as compared to laparotomy.
Proper positioning in left lateral tilt is important to reduce venal caval compression and maintain adequate placental blood flow, and open entry technique is recommended to prevent injury to the enlarged gravid uterus. Intraabdominal pressure with pneumoperitoneum should be limited to 10 to 12 mm Hg to reduce the theoretical concern of fetal acidosis associated with the effect of CO2.
There is no indication for intraoperative fetal monitoring.
Early surgery for appendicitis is also recommended in pregnant patients, as the consequences for both the mother and fetus can be catastrophic. Abbasi et al., in the largest case series to date involving 7000 patients, documents a markedly increased risk of severe complications with conservative management to include miscarriage and maternal sepsis. As with cholecystitis, non-operative management of appendicitis is contraindicated in pregnancy.
This patient undergoes an uncomplicated laparoscopic cholecystectomy. On postoperative day 2, she develops increasing pain in her right upper quadrant (RUQ) with fever and recurrent leukocytosis as well as elevated total bilirubin, transaminase, lipase and amylase levels. RUQ ultrasound documents dilated biliary ducts. Which of the following would be the most appropriate next step?
A. MRCP (magnetic resonance cholangio-pancreaticogram)
B. ERCP
C. Continued observation with antibiotic therapy
D. Repeat surgery with bile duct exploration
E. Delivery of the fetus
B.
Following cholecystectomy, this patient presents with findings consistent with choledocholithiasis, with associated gallstone pancreatitis and possible cholangitis.
The best option in this case is ERCP with the option of sphincterotomy to decompress the biliary tract. This approach appears safe in pregnant patients with early onset cholangitis with lower morbidity than conservative management.
In this patient, repeat surgery with intraoperative cholangiography or bile duct exploration would be a backup option, if the stones cannot be removed via ERCP.
Percutaneous biliary tract decompression would be another option in a high risk patient. MRCP is an excellent and safe diagnostic test in pregnancy and is a viable option in patients where the diagnosis is uncertain.
In a case with a high suspicion of cholangitis, this step could delay therapy which could have severe consequences for both fetus and mother.
More aggressive therapy for cholangitis is therefore indicated in pregnancy.
Although conservative treatment with continued intravenous antibiotics and observation may be appropriate in the non-gravid patient, the risks of this non-surgical approach are higher in the gravid patient and predispose her to grave complications.
Premature delivery of the fetus is not indicated for the treatment of biliary disease.
One year after an apparently uncomplicated cholecystectomy for chronic cholecystitis and cholelithiasis, a 37/F presents epigastric pain and jaundice of 2 wk duration. Ultrasonography reveals a dilated CBD along with dilated IHDs. To establish the nature of the obstruction, which diagnostic procedure is best:
a. ERCP
b. Percutaneous transhepatic cholangiography
c. CT scan
d. HIDA scan
a. ERCP
A 40/M from Davao presents with RUQ pain and fever. UTZ reveals a 6 cm complex mass in segment 7 of the liver. The treatment of choice is:
a. Percutaneous UTZ-guided aspiration
b. Laparoscopic surgery for drainage
c. Extra-peritoneal approach for drainage
d. Appropriate doses of metronidazole
d. Appropriate doses of metronidazole
A patient undergoes cholecystectomy, CBDE, T-tube choledochostomy for cholecystodocholithiasis. One week post-op, a T-tube cholangiogram shows multiple retained extrahepatic bile duct stones. Ideally this patient is best managed by:
a. choledochoscopic lithotripsy
b. re-exploration
c. extracorporeal shock wave lithotripsy
d. oral chenodeoxycholic acid
a. choledochoscopic lithotripsy
55/F consulted at the ER due to progressive painless jaundice, 1 day PTC, developed on and off of high grade fever associated with RUQ pain, on HBT-UTZ there was a cut off at the confluence of left and right IHDs, what will be your best treatment option at this time?
a. proceed with biliary exploration
b. ERCP with stenting
c. Bilateral percutaneous transhepatic biliary drainage
d. Conservative medical (antiobiotic / hydration) management
c. Bilateral percutaneous transhepatic biliary drainage
34/M underwent cholecystectomy, CBDE, t-tube choledochostomy. On follow up, the tube cholangiogram showed an impacted stone at the distal CBD. What is the best management for this patient?
a. flushing the t-tube with saline
b. choledochoscopy with stone extraction
c. ERCP, sphincterotomy with stone extraction
d. Re-exploration of the CBD
b. choledochoscopy with stone extraction
65/M came in at the ER due to abdominal pain and fever. On PE, the patient was noted to be jaundiced, tachycardic and hypotensive (70/40). Preliminary UTZ done at the ER showed dilated intrahepatic ducts. The best approach for this patient after resuscitation is:
a. Definitive surgery
b. Antibiotics alone
c. Immediate decompression
d. Conservative treatment
c. Immediate decompression
A 52-year-old male with a history of poorly controlled diabetes presents to the emergency department complaining of right upper quadrant pain and vomiting. An ultrasound is performed that shows gallstones with air in the gallbladder wall. Which of the following is the BEST next step in management of this patient?
a. IV antibiotics and cholecystectomy in the next 24-48 hours
b. Schedule elective cholecystectomy as an outpatient
c. ERCP
d. Emergent cholecystectomy
d. Emergent cholecystectomy
List the ligaments of the liver:
1) The falciform ligament,
2) coronary ligaments, and
3) the right and left triangular ligaments
What structures are contained within the hepatoduodenal ligament?
Porta hepatis:
1) proper hepatic artery
2) portal vein
3) common bile duct (CBD)
What line divides the liver into left and right lobes?
Cantlie line (runs from the middle of the gallbladder fossa anteriorly to the inferior vena cava posteriorly)
How many Couinaud segments are in the liver?
8
Which segments are removed in a right hepatectomy?
Segments 5 to 8
Which segments are removed in a right trisegmentectomy?
Segments 4 to 8
Which segments removed are in a left hepatectomy?
Segments 2 to 4
Which segments are removed in a left lateral trisegmentectomy?
Segments 2 and 3
Which segments are removed in a left trisegmentectomy?
Segments 2, 3, 4, 5, and 8
How much percentage of blood flow does the portal vein supply to the liver?
70%
What percentage of cardiac output accounts for hepatic blood flow?
25%
How much of the liver’s oxygen supply is provided by the hepatic artery?
50%
In Schwartz: The hepatic artery delivers approximately 25% of the blood supply, and the portal vein 75%.
How many hepatic veins are there?
Three (right, middle, left)
What is the approximate length of the CBD?
Approximately 7 cm
What is the normal CBD diameter?
0.5 to 1.5 cm. Depends on age and if gallbladder is still present.
Age: 0.2 cm to 1 cm (age above seventies).
If gallbladder removed, then CBD enlarges few mm.
What is the arterial supply of the bile ducts?
Primarily the right hepatic artery
Where does the sympathetic innervation of the gallbladder originate?
Celiac axis
What is the parasympathetic innervation of the gallbladder?
Vagus nerve
What are ducts of Luschka?
Accessory ducts directly from the liver bed into the gallbladder
How much bilirubin is produced daily in normal adults?
About 250 to 350 mg
How much bilirubin is produced from turnover of senescent red blood cells?
Approximately 85%
What is the most common cause of hyperbilirubinemia in adults?
Cholelithiasis
What is the major metabolite of heme?
Bilirubin
What is the name of sinusoidal macrophages?
Kupffer cells
What is the half-life of albumin?
20 days
What is the half-life of transferrin?
6days
Which major protein is produced by the liver?
Albumin
What are bile adds conjugated with to form bile salts?
Glycine and taurine
What is the major site of bile acid reabsorption?
Distal ileum
What autosomal recessive disorder causes an increase in conjugated bilirubin without elevation of liver function tests (LFTs)?
Dubin-Johnson syndrome
What percentage of the population has gallstones?
Approximately 10%
What group of Indians in southern Arizona are at high risk for gallstones?
Pima Indians
What population is at lowest risk for developing gallstones?
Sub-Saharan Africans
How are gallstones classified?
Cholesterol, black pigment, or brown pigment stones
What is the main composition of black pigmented stones?
Calcium bilirubinate
What is the main composition of brown pigmented stones?
Calcium salts of unconjugated bilirubin. Incidence increased with systemic infections.
What causes gallstone formation?
Cholesterol supersaturation, accelerated crystal nucleation, and gallbladder hypomotility
What are the 2 primary bile acids?
Cholate and chenodeoxycholate
What is the most common inherited cause of hyperbilirubinemia?
Gilbert syndrome (decreased activity of the enzyme glucuronyl transferase)
Which enzyme found in the cells of the bile duct rises with bile duct obstruction?
Alkaline phosphatase
The liver is the site of synthesis of all of the coagulation factors except:
von Willebrand factor
What is type I Crigler-Najjar syndrome?
Severe unconjugated hyperbilirubinemia, kernicterus, and bilirubin deposits in the brain causing severe motor dysfunction and retardation
What is type II Crigler-Najjar syndrome?
Less severe form, enzyme activity is 10% of normal
List the secondary bile salts:
Lithocholate, deoxycholate, and ursodeoxycholate
What hormone is a potent stimulator of gallbladder contraction?
Cholecystokinin
Study tip: CCK-HIDA is ordered for the workup of biliary dyskinesia.
EF <35% confirms the diagnosis of biliary dyskinesia.
Is prophylactic cholecystectomy recommended in diabetics?
No
What is the incidence of endoscopic retrograde cholangiopancreatography (ERCP)- induced pancreatitis?
About 5%
Which is a rapid, noninvasive imaging study that provides detailed biliary tree and pancreatic duct images equal to those of ERCP?
Magnetic resonance cholangiopancreatography (MRCP)
What percentage of pigmented stones and cholesterol stones are seen on plain abdominal films?
50% and 20%, respectively
What is the principal imaging modality for the diagnosis of cholelithiasis?
Ultrasonography
What is the sensitivity of ultrasonography for stones greater than 2 mm?
> 95%
What is the probability of complications requiring surgery in a patient with a history of biliary colic?
1% to 2% per year
What is the surgical management of uncomplicated biliary colic and documented gallstones?
Elective laparoscopic cholecystectomy
What percentage of patients with acute cholecystitis have positive enteric bacteria culture from the bile?
50%
What is hydrops of the gallbladder?
Chronic cystic duct obstruction and gallbladder distention with clear mucoid fluid
What is Mirizzi syndrome?
Impacted stone in the gallbladder neck causing extrinsic compression of the common (hepatic) duct
What is porcelain gallbladder?
Calcification of the gallbladder wall; cholecystectomy is indicated to prevent carcinoma.
Past estimates of gallbladder cancer in porcelain gallbladder were approximately 20%.
Recent reports suggest the estimate is lower than that, ranging between 7% and 15%.
What is Murphy sign?
Inspiratory arrest upon right upper quadrant (RUQ) palpation
What percentage of patients with gallstones have CBD stones?
15% to 20%
What is the most common cause of cholangitis?
Stone impacted in the CBD (85% of the time)
Which organisms are most commonly cultured from bile?
Escherichia coli, Klebsiella, Pseudomonas, Proteus, enterococci
What is the constellation of RUQ pain, jaundice, and fever?
Charcot triad
What is Reynold pentad?
Altered mental status and hypotension plus Charcot triad
Which autoimmune disease is associated with destruction of extrahepatic and intrahepatic bile ducts?
Primary sclerosing cholangitis
What is a type I choledochal cyst?
Saccular or fusiform dilatation of the common hepatic and CBD
What is a type II choledodial cyst?
This choledochal cyst is characterized as a diverticulum protruding from the CBD
What is a type III choledochal cyst?
Choledochocele; found in the intraduodenal portion of the CBD
What is a type IV choledochal cyst?
Choledochal cyst that involves the extrahepatic bile duct and intrahepatic ducts
What is a type V choledochal cyst?
Caroli disease (multiple intrahepatic cysts)
List the areas where dilated collateral veins are seen in patients with severe cirrhosis draining into the systemic circulation:
Hemorrhoidal, azygos, renal, adrenal veins, paraumbilical vein from abdominal wall
What is the most common cause of hemobilia?
Iatrogenic trauma to the liver and biliary tree
What is the first test to rule out hemobilia?
Esophagogastroduodenoscopy
What is the first-line therapy for hemobilia?
Angiography (embolization)
What is the treatment of pyogenic liver abscess?
Antibiotics and percutaneous drainage
What is the antibiotic course for pyogenic liver abscess?
Broad-spectrum intravenous antibiotics for 2 to 3 weeks followed by 4 to 6 weeks of oral antibiotics.
Duration of antibiotics is controversial, but most would give at least 2 weeks if no signs of bacteremia or sepsis.
What is the most common cause of hydatid disease?
Echinococcus granulosus
What cyst may rupture and result in anaphylactic shock?
Echinococcal cyst
What skin test is used in the diagnosis of hydatid disease?
Casoni test (intradermal injection of hydatid fluid)
What in the wall of a liver cyst is highly suggestive of hydatid disease?
Calcifications
What abnormality may be seen on complete blood count in about 25% of patients with echinococcal cyst?
Eosinophilia
What is the medical treatment of choice for Echinococcus?
Mebendazole/albendazole
What technique describes percutaneous management of hydatid cysts in patients who refuse or cannot undergo surgery?
PAIR (Percutaneous Aspiration, Injection, and Reaspiration)
What is the most common scolicidal agent used in hydatid cyst injection?
20% sodium chloride solution
What are surgical options for hydatid cysts?
Pericystectomy, partial hepatectomy
Cyst must be excised in whole without spilling its contents
What is the recurrence rate of postoperative hydatid cysts?
About 20%
What parasite causes amebiasis if transmitted through a few-oral route?
Entamoeba histolytica
What is the appropriate treatment of cyst rupture and bile duct obstrumon?
ERCP with papillotomy
What liver abscess is typically described as having an anchovy-paste appearance?
Amebic liver abscess
What is the treatment of amebic abscess?
750 mg of metronidazole 3 times a day for 10 days
What is the most common cause of intrahepatic presinusoidal hypertension worldwide?
Schistosomiasis
What is Budd-Chiari syndrome?
Hepatic venous outflow obstruction
What are some causes of Budd-Chiari syndrome?
Polycythemia vera (most common reason in Western part of the world), factor V Leiden mutation, thrombocytosis, protein C and S, antithrombin III, antiphospholipid antibody syndrome
What are possible sequelae ofhepatic adenomas?
Rupture or malignancy
What liver tumor is associated with oral contraceptive pill use?
Hepatic adenoma
How would you manage a patient with hepatic adenoma who wants to get pregnant?
Elective resection of hepatic adenoma
What liver mass is characterized by a central stellate scar seen on computed tomography (CT) imaging?
FNH
What is the indication for resection of FNH?
Symptomatic FNH or an enlarging lesion
What is Kasabach-Merritt syndrome?
Hepatic hemangioma, thrombocytopenia, and consumptive coagulopathy
What is the most common benign liver tumor?
Hemangioma
What are the 2 types of hepatic hemangioma?
Capillary hemangioma (clinically insignificant) and cavernous hemangioma
What diagnostic tests are indicated?
CT with IV contrast
Should biopsy be performed for a suspected hepatic hemangioma?
No (hemorrhage risk!)
Which hepatitis virus is transmitted by the fecal-oral route via contaminated food?
Hepatitis A
Which tests indicate acute hepatitis B virus (HBV) infection?
IgM anti-HBc, HBsAg
Which tests indicate chronic HBV infection?
HBsAg, IgG anti-HBc
What is caput medusae?
Dilation of paraumbilical veins arising from the left portal vein and extending to the umbilicus producing umbilical and abdominal wall varices
Where can collaterals form as a result of portal hypertension?
Esophagus, retroperitoneum, rectum, abdominal wall
What are the 3 categories of portosystemic shunts?
Nonselective, selective, and partial shunts
What are 2 basic types of nonselective shunts?
End-to-side portacaval shunt, side-to-side portosystemic shunt
What procedure has all but supplanted nonselective shunts?
Transjugular intrahepatic portosystemic shunt (TIPS)
What is the only commonly applied partial portosystemic shunt?
Interposition portacaval shunt
What is the surgical management of distal cholangiocarcinoma?
Pancreatoduodenectomy (Whipple)
What name is used to describe cholangiocarcinoma at the bifurcation of the common hepatic duct?
Klatskin tumor
What criteria for cholangiocarcinoma make it unresectable?
Bilateral hepatic artery involvement, encasement of the portal vein, bilateral hepatic duct involvement up to secondary radicals
What is the management of intrahepatic cholangiocarclnoma?
Hepatic resection
What is the management of perihilar cholangiocarcinoma?
Resection ofthe extrahepatic bile duct, cholecystectomy, and hepaticojejunostomy
Which potent hepatotoxin is produced by Aspergillus species?
Aflatoxin
What is the most common malignant hepatic tumor?
Metastases
What is the most common primary liver cancer worldwide?
Hepatocellular carcinoma (HCC)
What is the most common cause of HCC?
Hepatitis B and C
What distinct clinical variant of HCC is a well-circumscribed solitary lesion with a central scar?
Fibrolamellar carcinoma (better prognosis)
Which tumor marker may be helpful in the diagnosis of HCC?
a-Fetoprotein (AFP)
What are possible treatment options for HCC?
Partial hepatectomy, total hepatectomy with transplantation, ablation, embolization
This hepatic lesion does not produce AFP, but is associated with elevated neurotensin levels:
Fibrolamellar HCC
What is the most common primary hepatic tumor of childhood?
Hepatoblastoma
What hepatic sarcoma is associated with vinyl chloride, thorotrast, and arsenic?
Angiosarcoma
Which of the following lesions should be resected in the asymptomatic patient?
A. 5-cm FNH
B. 3-cm hepatic adenoma
C. 4-cm hydatid cyst
D. 6-cm hepatic hemangioma
E. 2-cm hepatic hamartoma
Answer: B.
Although regression of hepatic adenomas has been reported after discontinuation of oral contraceptives, the potential for bleeding and malignant transformation favors routine resection.
Which of the following is true regarding hemobilia?
A. Cholelithiasis is the most important cause of hemobilia.
B. Hepatoblastoma is the most commonly associated tumor.
C. It always presents with acute upper or lower gastrointestinal bleeding.
D. A tagged RBC scan is the diagnostic modality of choice.
E. Angioembolization is the initial definitive treatment of choice.
Answer: E.
Hemobilia is bleeding within the biliary tract. Causes include trauma, surgery, malignancy, and infections. Angiography is the most accurate and helpful diagnostic and therapeutic modality.
A 2-year-old boy is found to have fusiform dilation of the extrahepatic biliary duct. What is the best treatment option?
A. Transduodenal cyst excision
B. Excision with primary choledochorrhaphy
C. Complete cyst excision with Roux-en-Y reconstruction
D. Liver transplantation
E. Observation
Answer: C.
Fusiform extrahepatic dilation is a type I choledochal cyst.
Surgical resection is necessary to avoid recurrent episodes of infection from stasis of the bile within the cyst cavity, as well as the risk of developing a cholangiocarcinoma.
Type III cysts usually do not require resection.
Types I and II do have a cancer risk.
Therefore, the extrahepatic bile duct is resected and a Roux-en-Y choleenterostomy is performed.
What is the best treatment for a 6-cm encapsulated cyst of the right lobe of the liver in a febrile patient?
A. Right hepatectomy
B. Percutaneous drainage
C. Marsupialization
D. Albendazole
E. Metronidazole
Answer: B.
Pyogenic liver abscesses are best treated with initial drainage and broad-spectrum antibiotics.
Albendazole is reserved for hydatid/echinococcal cysts, while metronidazole is used for amebic cysts.
Resection and marsupialization are not first-line treatments of benign liver cysts.
Which of the following methods is the most accurate for detection of hepatic metastases?
A. Intraoperative palpation
B. CT
C. Laparoscopy
D. Transabdominal ultrasound
E. lntraoperative ultrasound
Answer: E.
Performed with handheld or laparoscopic transducers, intraoperative ultrasound has been established as the most accurate method to detect hepatic metastases.
It is a useful modality for identifying vascular structures and their relation to liver lesions, hence enabling an evaluation for resectability.
CT scanning is accurate for larger lesions, usually measuring more than 2 cm.
A 55-year-old man undergoes a laparoscopic cholecystectomy for acute cholecystitis. The final pathology report indicates gallbladder carcinoma that invades the muscularis. Which of the following would be the most appropriate management?
A. Radiation
B. Chemotherapy
C. Combined chemotherapy and radiation therapy
D. Observation
E. Reoperation with wedge resection of liver around the gallbladder fossa with regional lymph node dissection
Answer: E.
Gallbladder adenocarcinoma that is classified as carcinoma in situ or T1a (invades lamina propria) and that has negative margins if discovered incidentally
and removed can be managed by cholecystectomy alone.
Lesions that are T1b (invades muscularis propria layer), T2 (invades the perimuscular connective tissue that is superficial to the muscularis propria), or greater are treated with a radical cholecystectomy, which consists of subsegmental resection of liver segments 4b and 5, in addition to a hepatoduodenal ligament lymphadenectomy and cystic duct excision if the margin is positive.
Remember that the gallbladder has no submucosa.
Adjuvant chemotherapy and chemotherapy for unresectable disease can be considered.
A 25-year-old man presents with high fever and RUQ pain. His WBC count is 19,000. An ultrasound shows a 6-cm fluid collection in the right lobe of the liver. On CT scan, the fluid collection shows a peripheral rim of edema. The cause of the fluid collection is most likely determined by:
A. Blood and stool cultures
B. Percutaneous aspiration of the liver lesion
C. Serologic tests
D. LFTs
E. MRCP
Answer: C.
Amebic liver abscesses are caused by E. histolytica. Diagnosis is made by clinical presentation, ultrasound, and, most notably and distinctly, by CT or serologic testing.
The classic finding on CT is a single fluid collection in the right lobe with a rim ofperipheral edema, which distinguishes this entity from pyogenic and echinococcal cysts.
A cirrhotic patient presents with asymptomatic umbilical hernia with overlying erythematous skin changes. What is the most appropriate management?
A. Observation and delayed repair
B. Repair with primary fascial closure
C. Repair with synthetic permanent mesh
D. Repair with biologic mesh
E. Therapeutic paracentesis
Answer: C.
A recent Dutch study suggests that after excluding patients with an intact umbilical vein, umbilical hernia repair can be performed safely in patients with liver cirrhosis.
The authors point out that the ideal time to repair such hernias would be at the time of transplantation.
However, for many patients, the waiting time prior to transplantation can be prolonged, increasing the likelihood of an incarcerated hernia developing. Thus, it may be prudent to perform herniorrhaphy on some of these patients while they are awaiting liver transplantation.
Moreover, a permanent mesh can be used in complicated hernias in cirrhotic patients with minimal wound-related morbidity and a significantly lower rate of recurrence.
A 38-year-old woman presents with RUQ pain. Imaging with CT shows a 3-cm mass in segment 7 of the liver that is well demarcated with a central fibrotic area. This central scar does not enhance in the arterial phase of the CT scan. Workup shows no evidence of cirrhosis, hepatitis and a normal AFP level, and slightly elevated neurotensin. What is the diagnosis?
A. FNH
B. Hepaticadenoma
C. Fibrolamellar carcinoma
D. Hepatoma
E. Hemangioma
Answer: C.
The fibrolamellar variant of HCC (or hepatoma) is now sometimes considered a distinct pathologic entity.
It occurs in younger patients.
Unlike HCC, most patients are not cirrhotic, not hepatitis B positive, and have normal AFP levels and may have elevated neurotensin levels.
They tend to have a better prognosis than HCC.
The tumor is well demarcated and generally has a central fibrotic area.
As opposed to the central scar seen in FNH, this is not a vascular lesion and therefore will not enhance in the arterial phase of the CT, nor with gadolinium-enhanced MRI, whereas FNH will.
The Model for End-stage Liver Disease (MELD) score is based on what 3 parameters?
A. International normalized ratio {INR), creatinine, bilirubin
B. Albumin, creatinine, alkaline phosphatase
C. INR, bilirubin, AST
D. Amylase, GGT, albumin
E. AST, ALT, alkaline phosphatase
Answer: A.
The MELD is a scoring system for assessing the severity of chronic liver disease. It was initially developed to predict death within 3 months of surgery in patients who had undergone a TIPS procedure and was subsequently found to be useful in determining the prognosis and prioritizing for receipt of a liver transplant.
This score is now used by the United Network for Organ Sharing and Eurotransplant for prioritizing allocation of liver transplants instead of the older Child-Pugh score.
It is based on the INR, creatinine, and bilirubin, thereby avoiding the inclusion of subjective parameters that are incorporated in the previously used Child-Pugh score.
More recently, a modified MELD score has been used. The modified score incorporates sodium level into its calculation.
What is the first step to clear a CBD stone found on an intraoperative cholangiogram?
A. Transcystic duct exploration
B. Administer IV glucagon and flush duct with saline
C. Finish the laparoscopic cholecystectomy with plans for postoperative ERCP
D. Open CBD exploration
E. Choledochotomy and T-tube insertion
Answer: B.
Flushing the biliary tree with saline through the cystic duct and administering IV glucagon frequently helps in clearing the CBD of stones, especially if they are small.
Glucagon relaxes the sphincter of Oddi, which helps in clearing the stones.
If that fails, then the next step would be passing a Fogarty catheter or a basket under fluoroscopy for stone retrieval.
If that fails, then a choledoscope can be inserted along with a basket or wire catheter.
If that fails, then a choledochotomy is made and again a choledoscope can be used, or open exploration is commenced.
Which of the following demonstrates a peripheral-to-central enhancement on CT with IV contrast?
A. Focal nodular hyperplasia
B. Adenoma
C. Hemangioma
D. HCC
E. Hepatic metastases
Answer: C.
Hemangiomas have peripheral to central enhancement on CT scan.
The centripetal enhancement can be seen on the portal venous phase of the CT scan.
FNH has a central stellate scar.
Adenomas appear as a well-circumscribed mass and have homogenous enhancement on the arterial phase.
Which of the following is a correct statement?
A. The cystic artery is most commonly a branch of the left hepatic artery.
B. The blood supply for the CBD enters at the 12 and 6 o’clock positions.
C. The portal vein is the major venous drainage system of the liver.
D. The CBD lies posteriorly in the portal triad.
E. Choledochotomy is done vertically.
Answer: E.
The CBD lies lateral to the hepatic artery proper and anterior to the portal vein in the portal triad.
Its blood supply enters the duct at the 3 and 9 o’clock positions.
So a choledochotomy is done vertically to avoid injuring the blood supply of the CBD.
The cystic artery is most commonly a branch of the right hepatic artery.
The portal vein does not drain the liver; on the contrary, it supplies blood to the liver.
Which of the following statements regarding metastases to the liver is true?
A. The hepatic artery is the major blood supply of the metastases.
B. The most common primary cancer is lung.
C. Primary liver cancers are more common than metastases.
D. Resection of colorectal cancer metastasis to the liver does not improve survival.
E. Resection of metastasis to the liver is recommended only if there are fewer than 2 metastases and each is smaller than 3 cm
Answer: A.
Metastasis to the liver is fairly common with multiple tumors, including colorectal, pancreatic, cholangiocarcinoma, lungs, and breast.
It is more common than primary liver cancers.
The most common primary cancer is colorectal.
Resection of colorectal metastases to the liver improves 5-year survival to approximately 30%. The blood supply of the metastasis is from the hepatic artery.
This is clinically significant in certain treatment modalities such as chemoembolization.
Metastasectomy is recommended regardless of size and number of metastases as long as the remaining liver is functional (>20%).
A 42-year-old immigrant presents to the ED with right upper quadrant pain. Workup reveals a large cyst occupying his right liver with a calcified wall and multiple small other cysts in the same lobe. Which of the following is the most appropriate therapy?
A. Albendazole only
B. Marsupialization of the cyst
C. Right hepatectomy and adjuvant chemotherapy
D. IR drainage of the largest cyst
E. Aspiration of the contents, injection of hypertonic saline, followed by cystectomy
Answer: E.
This patient is presenting with hydatid disease. It is due to E. granulosus in most instances.
The therapy is mainly surgical, including resection of the cyst after aspirating its contents and injecting it with a scolicidal agent to kill the scolices.
Agents include hypertonic saline.
Towels should be placed around the liver to minimize the risk of anaphylaxis in case of spillage.
There is no role for chemotherapy or antihelminthics alone.
Formal resection is aggressive and not required.
Which of the following statements about the anatomy of the liver is true?
A. The right and left lobe are divided by the falciform ligament.
B. The portal triad consists of the hepatic artery, hepatic vein, and bile duct.
C. The caudate lobe has its own separate arterial supply.
D. The right lobe contains anterior and posterior segments.
E. The American system describes eight segments.
ANSWER: D
COMMENTS: The surgical anatomy of the liver is based on the distribution of the hepatic veins and portal structures, and it has been modified several times.
There are two main anatomic classification systems for the liver: the American system and the French system.
In both these systems, the liver is divided into right and left lobes by the Cantlie line, a longitudinal plane that extends from the gallbladder fossa to the inferior vena cava. This plane, also called the portal fissure, contains the middle hepatic vein and the bifurcation of the portal vein.
In the American system, the liver is further broken down into four segments, with each lobe containing two segments. The right lobe of the liver consists of posterior and anterior segments. The left lobe consists of a medial segment (quadrate lobe) and a lateral segment divided by the falciform ligament. The caudate lobe can be considered anatomically independent of the right and left lobes because it receives portal and arterial blood supply from both sides and has venous drainage directly into the inferior vena cava.
In the French system, developed by C. Couinaud, the two lobes of the liver are broken down into eight segments. These eight segments are formed by three vertical planes (scissurae) created by the right, middle, and left hepatic veins, which results in four sectors. These four sectors are further divided by a plane created by the branching portal system. Therefore the left lobe, according to the French system, is divided into medial and lateral segments by the left hepatic vein. The lateral sector of the left lobe consists of a superior segment (II) and an inferior segment (III). The medial sector of the left lobe is segment IV. The right lobe consists of anteromedial and posterolateral sectors divided by a vertical plane containing the right hepatic vein. The anteromedial sector is made up of segment V (inferior) and segment VIII (superior), and the posterolateral sector is made up of segment VI (inferior) and segment VII (superior).
Which of the following statements is true about the hepatic arterial supply?
A. Aberrant hepatic arterial anatomy is present in less than 5% of all patients.
B. The cystic artery is usually a branch off the proper hepatic artery.
C. A “replaced” right hepatic artery arises from the superior mesenteric artery.
D. The hepatic artery provides 75% of the blood flow to the liver.
E. The hepatic artery lies dorsal to the portal vein within the hepatic hilum.
ANSWER: C
COMMENTS: The hepatic arterial supply is normally derived from the celiac axis by way of the common hepatic artery, which becomes the proper hepatic artery after giving off the gastroduodenal branch and subsequently bifurcates into right and left hepatic branches.
The hepatic artery lies ventral to the portal vein. The middle hepatic artery is usually a branch off the left hepatic artery, and the cystic artery is generally a branch off the right hepatic artery.
There is, however, significant variability in hepatic arterial anatomy in up to 50% of patients. In approximately 15% of individuals, the right hepatic artery arises from the superior mesenteric artery (replaced right hepatic artery) and is found in the right dorsal border of the hepatoduodenal ligament. In roughly 10% of individuals, the left hepatic artery originates from the left gastric artery and is located in the gastrohepatic ligament. These commonly encountered variants can have important surgical implications during upper abdominal operations. The arterial blood supply accounts for only 25% of hepatic blood flow, with the remainder being supplied by the portal vein.
Ref.: 1–3
Which of the following statements about the anatomy of the hepatic veins is true?
A. The left hepatic vein drains the entire left lobe.
B. Veins from the caudate lobe enter both the left and middle hepatic veins.
C. The middle hepatic vein usually joins the left hepatic vein.
D. There are valves in the hepatic venous system.
E. Hepatic veins have prominent hyperechoic walls on ultrasound imaging.
ANSWER:
C
COMMENTS: The hepatic veins begin in the liver lobules as the central veins and coalesce to form the right, left, and middle hepatic veins, which drain into the inferior vena cava and are of considerable surgical importance because they define the three vertical scissurae of the liver.
The right vein, which is generally the largest, drains most of the right lobe. The left vein drains the lateral segment of the left lobe and a portion of the medial segment as well. The middle vein drains the inferoanterior portion of the right lobe and the inferomedial segment of the left lobe. This vein joins the left hepatic vein in 80% of individuals and enters the inferior vena cava directly in the remainder.
There are also smaller veins, particularly those draining the caudate lobe dorsally, that enter directly into the inferior vena cava. The human hepatic venous system has no valves. The portal veins and hepatic veins can readily be differentiated from each other on the basis of their distinctive sonographic features. The portal veins (not the hepatic veins) have prominent hyperechoic walls.
- Which of the following statements is true about the portal vein?
A. It is formed by the junction of the inferior mesenteric vein and splenic vein.
B. It is the most dorsal structure in the hepatoduodenal ligament.
C. It contains the valves of Mirizzi.
D. The right portal vein typically branches later than the left portal vein.
E. It carries deoxygenated blood and provides only 10% of the liver’s oxygenation.
ANSWER:
B
COMMENTS: The portal vein is usually formed dorsal to the neck of the pancreas by the junction of the superior mesenteric vein and splenic veins. It ascends posterior to the common bile duct and hepatic artery in the hepatoduodenal ligament. These three structures make up the portal triad. There are no valves in the portal venous system (Pablo Mirizzi described valves in the common hepatic duct that do not exist). The portal vein bifurcates just outside the liver. The right portal vein has anterior and posterior branches that typically diverge only a short distance from the bifurcation and then quickly dive into the liver parenchyma. The left portal vein has a longer transverse portion (pars transversus) and then angulates anteriorly in the umbilical fissure (pars umbilicus), where it gives off medial branches to segment IV and lateral branches to segments II and III. The portal vein provides approximately 75% of the hepatic blood flow, and although the blood is largely deoxygenated, it provides up to 50%–70% of the liver’s oxygenation secondary to the portal system’s large volume flow rate.
Which of the following characteristics is typically seen on ultrasound imaging of the hepatic portal vein branches?
A. Hyperechoic vessel walls
B. Hepatofugal blood flow
C. Diastolic reversal of blood flow
D. Location between hepatic segments
E. Vertical orientation
ANSWER: A
COMMENTS: The portal veins and hepatic veins can readily be differentiated from each other on the basis of their distinctive sonographic features. The portal vein and its branches have prominent hyperechoic walls. This appearance has been attributed to the accompanying intrahepatic branches of the hepatic artery and bile duct, which are not generally seen individually on external ultrasound imaging. In contrast, the hepatic veins appear to be essentially “wall less.” They are anechoic or hypoechoic tubular structures that are vertically oriented and increase in caliber as they course toward the inferior vena cava. The portal veins are more transversely oriented and of larger caliber centrally. The portal vein branches are located within the anatomic liver segments, and the hepatic veins are found between the segments. Doppler ultrasound permits characterization of flow patterns in the hepatic vessels. Under normal circumstances, portal vein flow is toward the liver (hepatopedal). Flow in the portal vein is usually of fairly low velocity, with minor undulations and continued forward flow during diastole. Flow in the hepatic veins is hepatofugal and varies according to the cardiorespiratory cycle. The portal veins are horizontally oriented, whereas the hepatic veins are vertically oriented.
Which of the following is true regarding the hepatic functional unit?
A. The center of the hepatic lobule is the hepatic venule.
B. Blood flows from the hepatic vein to the portal triad.
C. Zone I is the most susceptible to hypoxic injury.
D. Hepatocytes in zone I have the lowest oxygen tension.
E. Bile flows toward the centrilobular hepatic venule.
ANSWER: A
COMMENTS: The functional histologic unit of the liver is the acinus. At the center of the acinus is the portal triad, which consists of a terminal branch of the portal vein (portal venule) along with a hepatic arteriole and bile ductule. Blood from the terminal portal venule goes into the hepatic sinusoids, around which hepatocytes are located. Eventually, blood returns to the central vein leading to the terminal hepatic venules at the periphery of the acinar unit. The hepatocytes of the acinus are divided into three zones, with zone I being closest to the afferent portal venule and zone III being nearest the efferent central hepatic venule. Zone II is between these two points. Within the acinus, there is a gradient of solute concentration and oxygen tension that is greatest near the portal venules at the center of the acinus. The hepatocytes in zone I are therefore exposed to more oxygen and are less subject to hypoxia compared with the hepatocytes near the periphery of the acinus (zone III). This explains the histologic pattern of centrilobular necrosis that occurs following ischemia. The hepatic venule is at the center of the histologic hepatic lobule. Each hepatic lobule is thus surrounded by several peripheral acini. Bile is formed within the hepatocytes and empties into terminal canaliculi, which coalesce into bile ducts. The bile then flows toward the portal triad.
What is the pathologic feature of cirrhosis?
A. Apoptosis of hepatocytes, involving lobules and portal
tracts
B. Fat accumulation in hepatocytes
C. Brown pigment in hepatocytes
D. Periductal fibrosis with an onion-skin appearance
E. Dense matrix material deposition in the perisinusoidal space
ANSWER: E
COMMENTS: Cirrhosis may be due to many processes. It is defined as end-stage liver damage with hepatocyte death and dis- ruption of hepatic parenchyma by diffuse fibrosis (dense matrix material deposition) and abnormal nodular architecture. Pathologic characteristics include fibrous septa around regenerative nodules of hepatocytes.
The bands of fibrosis originate from stellate cells located beneath endothelial cells that line the sinusoids, in the space of Disse (perisinusoidal space).
Clinical manifestations include increased hepatic pressure, portal venous congestion and portal hypertension, ascites, coagulopathy, steroid hormone imbalances, encephalopathy, hepatorenal syndrome, and hepatopulmonary syndrome.
Despite historical schools of thought, it is now known that even in late-stage disease, some regression of cirrhosis is possible.
Apoptosis of hepatocytes involving lobules and portal tracts is seen in hepatitis.
Fat accumulation in hepatocytes is seen in the alcoholic fatty liver disease.
Brown pigment in hepatocytes, or hemosiderosis, is due to iron deposition in hepatocytes in hemochromatosis.
Periductal fibrosis with an onion-skin appearance is seen in primary biliary cirrhosis.
Which of the following statements related to liver embryology is true?
A. The liver is derived from the hindgut.
B. Two umbilical veins carry placental blood to the liver.
C. One umbilical artery carries placental blood to the placenta.
D. The ductus venosus shunts blood from the portal vein to the systemic circulation.
E. The falciform ligament is the obliterated ductus arteriosus.
ANSWER: D
COMMENTS: The liver, gallbladder, and biliary tree are derived from a ventral diverticulum of the junction of the foregut and midgut.
In fetal circulation, one umbilical vein carries oxygenated blood from the placenta to the ductus venosus, which shunts blood from the portal vein to the inferior vena cava. The result is that the maternal blood, which has already undergone hepatic metabolism in the mother, largely bypasses the fetal liver. Two umbilical arteries carry deoxygenated blood back to the placenta. In the transition to extrauterine circulation, the umbilical vein degenerates as flow ceases, and stasis and thrombosis cause ductus venosus closure.
In the adult, the falciform ligament carries the umbilical vein remnant from the umbilicus. The ligamentum teres extends from the falciform ligament and carries the obliterated umbilical vein to the undersurface of the liver. The ligamentum venosum derives from the ductus venosus.
During fasting, the liver provides energy substrates by all but which of the following mechanisms?
A. Glycogenolysis B. Glycolysis C. Gluconeogenesis from alanine D. Gluconeogenesis from lactate E. Formation of ketone bodies from fatty acids
ANSWER: B
COMMENTS: The liver plays a pivotal role in energy metabolism. In the fed state, glucose is converted to glycogen for storage. The liver itself obtains its energy primarily from ketoacids rather than glucose, although it can use glycolysis during periods of glucose excess (fed state).
During fasting, the liver provides glucose by the breakdown of the stored glycogen (glycogenolysis). Glucose is a critical energy source for red blood cells, the central nervous system, and the kidneys. Because glycogen stores are depleted after about 48 h, the liver generates glucose from other sources.
Alanine, other amino acids, lactate, and glycerol can serve as carbon sources for gluconeogenesis. Lipolysis occurs during prolonged fasting, and the fatty acids released from adipose stores are oxidized in hepatocytes to form ketone bodies.
Ketone bodies are an important alternative fuel source for brain and muscle.
The cytochrome P-450 system transforms compounds by all except which of the following mechanisms?
A. Oxidation B. Hydrolysis C. Conjugation D. Reduction E. Hydrogenation
ANSWER: C
COMMENTS: The liver is responsible for the biotransformation of many endogenous and exogenous substances. For the most part, this process detoxifies potentially injurious substances and facilitates their elimination.
In some instances, however, hepatic bio- transformation produces more toxic metabolites.
There are two general mechanisms by which the liver accomplishes biotransformation: oxidation, reduction, and hydrolysis (phase I reactions) and conjugation (phase II reactions).
The cytochrome P-450 enzyme system catalyzes phase I reactions. The second mechanism involves an array of enzymes that conjugate substances with other endogenous molecules.
These reactions are referred to as phase II reactions, and their purpose is to convert hydrophobic compounds to hydrophilic ones that are water soluble and can thus be eliminated in bile or urine.
The liver is also the principal site of conversion of ammonia to urea via the urea cycle, which is a separate process.
A 63-year-old woman with unresectable cholangiocarcinoma undergoing palliative therapy presents with fever, right upper quadrant pain, jaundice, and cough for 1 day. Temperature was 101.7°F, pulse 93 beats/min, blood pressure 112/65 mmHg, respiratory rate 20 breaths/min, and saturation 98% on room air. Physical examination shows a soft and nondis- tended abdomen with tenderness in the right upper quadrant and hepatomegaly. Lab results include a white blood cell count of 14,000/mm3 and alkaline phosphatase level of 215 IU/L. Chest x-ray shows an elevated right hemidiaphragm and right lower lobe atelectasis. Ultrasound is performed showing a round hypoechoic lesion within the liver, with well-defined borders and several internal echoes, mildly dilated intrahepatic bile ducts, and no cholelithiasis or pericholecystic fluid. In addition to further imaging, what is the most appropriate management?
A. Antibiotics for 10 to 14 days
B. Percutaneous drainage and intravenous (IV) antibiotics C. Endoscopic retrograde cholangiopancreatography
D. Laparoscopic cholecystectomy
E. Emergent exploratory laparotomy
ANSWER: B
COMMENTS: Liver abscesses classically present as fever, right upper quadrant pain, nausea, pleuritic chest pain, cough or dyspnea, and sometimes with sepsis. They may be pyogenic (bacterial), amebic, or fungal. Pyogenic abscess is usually polymicrobial, although common microbes include Escherichia coli or other gram- negative bacteria, Streptococcus species, and anaerobes such as Bacteroides.
Patient risk factors include underlying biliary cancer, recent ablative liver therapy, or liver transplantation. Today, the most frequent source of pyogenic abscess is a contiguous infection in the biliary tract, such as cholangitis. Other sources include infectious foci within the portal venous drainage system, direct extension from perihepatic sites, and hematogenous spread. The right lobe is the most commonly involved, which has been attributed to a streaming effect on the portal vein. Approximately 20% of pyogenic abscesses are cryptogenic.
The diagnosis is based on the clinical findings and hepatic imaging and may be confirmed by fine-needle aspiration. While ultrasound is a good modality to visualize the liver abscess, gallbladder, and intrahepatic bile ducts, computed tomography (CT) is more sensitive and further allows for evaluation of another underlying cause such as a biliary tree cancer.
Treatment of pyogenic abscess requires eradication of both the abscess and the source. Treatment of the abscess usually requires drainage by operative or percutaneous approaches. Antibiotic therapy alone may suffice for the treatment of multiple small abscesses.
A 30-year-old man visiting from Mexico comes to the emergency department with a history of 2 weeks of right upper quadrant pain and tenderness, fevers, chills, and diarrhea. He is febrile to 102.9°F. His heart rate and blood pressure are 120 beats/min and 100/75 mmHg, respectively. Laboratory results include a white blood cell count of 16,000/mm3, aspartate aminotransferase (AST) level of 50 IU/L, and alanine aminotransferase (ALT) level of 93 IU/L. Ultrasound of the abdomen shows a 4 × 7-cm2 round, hypoechoic, nonhomogeneous lesion abutting the liver capsule without rim echoes. Subsequent CT also demonstrates a non–rim-enhancing hypoechoic lesion with a smaller adjacent lesion measuring 2 × 2 cm2. Which of the following is the most appropriate course of action?
A. Observation
B. Open surgical drainage
C. Broad-spectrum antibiotics and percutaneous drainage
D. Serologic testing for Entamoeba histolytica and oral metronidazole
E. Therapeutic fine-needle aspiration
ANSWER: D
COMMENTS: Amebic abscesses are caused by the protozoan E. histolytica, which is spread through the fecal–oral route. Once ingested, the cysts pass into the intestines, where the trophozoite is released and transmitted to the colon. These trophozoites can then invade the colonic mucosa and subsequently reach the liver via the portal vein. In the liver, these trophozoites produce a liquefaction necrosis responsible for the classic “anchovy paste” appearance. Protozoa are not usually isolated from the abscess because they are located in the peripheral rim of tissue.
Diagnosis requires hepatic imaging (usually ultrasound or CT) and serologic testing for the presence of E. histolytica antibodies as well as a thorough history and physical examination. The patient in this question is a young man from an endemic region who has signs and symptoms similar to those of a pyogenic liver abscess; however, his classic history and the lack of rim enhancement on imaging suggest the diagnosis of amebic abscess rather than a pyogenic abscess.
Hepatic amebiasis is treated primarily by the administration of amebicidal drugs, with metronidazole being the drug of choice. Percutaneous aspiration may be indicated if the patient does not respond to medical management or the diagnosis is in question. Percutaneous or operative drainage is also indicated in the presence of secondary bacterial infection, which occurs in about 10% of amebic abscesses.
A 50-year-old woman complains of a 4-month history of right-sided abdominal pain and nausea. Her vital signs are stable, and she is afebrile. Her physical examination is unremarkable except for hepatomegaly. Ultrasound of the abdomen shows an 8-cm well-circumscribed cyst with a rosette appearance. What is the preferred treatment of this patient?
A. Pericystectomy B. Percutaneous catheter drainage C. Transperitoneal surgical drainage D. Metronidazole E. Albendazole
ANSWER: A
COMMENTS: The helminth Echinococcus granulosus is respon- sible for most hydatid diseases of the liver. It is usually a unilocular process involving the right lobe, although it may be manifested as multiple cysts. Complications include intrabiliary, intraperitoneal, or intrapleural rupture; secondary infection; anaphylaxis; and mass replacement of the liver.
These lesions often have a calcified wall and can be diagnosed serologically by indirect hemagglutination tests, complement fixation tests, serum immunoelectrophoresis, and, formerly, the Casoni skin test.
CT and ultrasound may demonstrate characteristic daughter cysts (hydatid sand) or grand- daughter cysts (rosette appearance) within the cyst.
Treatment is primarily surgical. Percutaneous aspiration or drainage is generally contraindicated because of the risk for intraperitoneal dissemination; however, since the advent of chemotherapeutic agents such as albendazole, some clinicians have proposed percutaneous drainage.
The principles of surgical therapy are to avoid spillage and remove the entire germinal layer. The cyst consists of an inner germinal layer (endocyst) and an outer fibrous membrane layer (pericyst).
Resection is usually accomplished by pericystectomy. Anatomic hepatic resection is not generally required but may be used.
Surgery, in addition to preoperative and postoperative benzimidazole compounds, has been shown to be very effective.
Metronidazole is used for the treatment of amebic liver abscesses. Because 20% of echinococcal cysts exhibit biliary communication, assessment by preoperative ERCP or intraoperative cholangiography is important in any patient with jaundice, cholangitis, elevated liver enzyme levels, or bile noted during resection.
Scolicidal agents should be used with caution because of the risk of sclerosing the bile ducts in the event that the agent finds its way into the ductal system.
A 28-year-old asymptomatic white woman is incidentally found to have a 3.5-cm hypervascular lesion with a central scar in the right lobe of her liver. On delayed images, there is increased uptake of contrast material in the scar in compari- son with the surrounding liver parenchyma. She is otherwise healthy and takes no medications. Liver enzyme and α-fetoprotein levels are within normal limits. Which of the following is the most appropriate management of this patient?
A. Open liver resection B. Open surgical biopsy C. Observation D. Chemoembolization E. Hepatic artery embolization
ANSWER: C
COMMENTS: This patient has focal nodular hyperplasia (FNH), which is often found incidentally on imaging or during laparotomy.
FNH is a benign liver tumor that predominantly occurs in women in the third to fifth decades of life. It is similar to hepatic adenoma (HA) but with important differentiating clinical and histologic features and therapeutic implications.
Both occur most commonly in women of childbearing age; however, HA is associated with the use of oral con- traceptives and anabolic steroids and is also seen in certain glycogen storage diseases. HA is usually symptomatic (80% of cases) and is associated with rupture and bleeding in a substantial proportion of patients, whereas FNH is usually asymptomatic and found incidentally.
Furthermore, HA has the potential for malignant transformation, whereas the risk for malignancy in FNH is unlikely but uncertain.
Histologically, HA consists of hepatocytes without bile ducts or Kupffer cells. FNH contains Kupffer cells along with a central stellate scar surrounded by fibrous tissue. Scanning for Kupffer cell activity with technetium-99m (99mTc)-labeled sulfur colloid is thus useful in differentiating the lesions. Because of the asymptomatic nature of this patient, small size of the lesion, and negligible risk for malignant transformation, observation is appropriate.
Surgical resection is reserved for symptomatic patients or when the diagnosis is uncertain.
Right upper quadrant abdominal pain develops in a 25-year- old woman taking oral contraceptives. CT demonstrates a hypodense, 6-cm mass in the right lobe of the liver. A 99mTc-labeled scan reveals a defect in the area of the mass. Angiography reveals a hypervascular tumor with a peripheral blood supply. Which of the following is the appropriate management?
A. Discontinuation of oral contraceptives and observation with serial CT B. Percutaneous needle biopsy C. Hepatic resection D. Arterial embolization E. Radiation therapy
ANSWER: C
COMMENTS: The imaging characteristics described are typical of HA. Because HA does not contain Kupffer cells, it does not take up radioisotope. This point may be useful for differentiating HA from FNH but not necessarily from other mass lesions of the liver.
Percutaneous biopsy of suspected HA is not advisable because of the risk for hemorrhage. HAs associated with oral contraceptives tend to be larger and have a higher risk for bleeding.
Regression does not reliably occur with cessation of oral contraceptives. However, for lesions smaller than 4 cm, a trial of cessation of contraceptives or steroids with observation may be attempted.
Resection is indicated for most suspected HAs, particularly for symptomatic lesions, for patients not taking oral contraceptives, and if the diagnosis is uncertain.
Embolization may be useful for treating hemorrhage in a patient whose HA is inoperable. Radiation has no role in the management of HA.
A 75-year-old woman with recurrent breast cancer undergoes CT of the chest, abdomen, and pelvis for metastatic workup. She is found to have a 7-cm liver mass. Follow up triple- phase CT shows a hypervascular lesion with peripheral to central enhancement. It is thought to be a hemangioma. She denies abdominal pain or any episodes of jaundice. What would be a reasonable indication for operative intervention?
A. Pain, shortness of breath, early satiety
B. Hemorrhage
C. High-output cardiac failure
D. Disseminated intravascular coagulation (DIC)
E. All of the above
ANSWER: E
COMMENTS: Hepatic hemangiomas are the most common benign hepatic tumor. They have no risk of malignant degeneration and are often found incidentally. They typically appear as hyper- vascular lesions with initial peripheral enhancement followed by late central enhancement, which follows the direction of blood flow into the lesion. Biopsy is not necessary and furthermore carries the risk of hemorrhage.
Asymptomatic uncomplicated hemangiomas are simply observed. Indications for surgical resection include symptoms from mass effect, consumptive coagulopathy manifesting as DIC (seen in Kasabach-Merritt syndrome), high-output cardiac failure from arteriovenous shunting, or rupture and hemorrhage. Enucleation can generally be performed, without formal liver resection being necessary.
A 50-year-old woman with hypertension presents to her primary care physician (PCP) with right upper quadrant pain and jaundice. There is a palpable soft mass along her liver edge. Workup is negative for viral hepatitis, and liver enzymes are mildly elevated. Right upper quadrant ultra- sound shows a 4-cm cystic structure that is extrahepatic and separate from the gallbladder. ERCP is performed and demonstrates a biliary diverticulum obstructing the common bile duct. What is the most appropriate next step in management?
A. Observation B. Percutaneous drainage C. Cholecystectomy D. Resection E. Hepaticojejunostomy
ANSWER: D
COMMENTS: When a cyst is an incidental finding with no symptoms and the diagnosis is secure, no further intervention is indi- cated. In larger cysts, there is a risk of biliary obstruction leading to pancreatitis, cholangitis, and obstructive jaundice, and symptom- atic patients should undergo resection.
Percutaneous drainage or injection of alcohol or other sclerosing agents does not suffice and is not recommended. If a cyst is found to communicate with the bile ducts, either excision or Roux-en-Y cystojejunostomy may be performed.
Choledochal cysts are classified into five types. Type I cysts are saccular dilatations of the entire common bile duct, and treat- ment is Roux-en-Y hepaticojejunostomy. Type II cysts, like in this case, are true diverticula of the common bile duct, and treatment is simple excision. Type III cysts, also called choledochodeles, are local dilations of the distal common bile duct extending into the duodenal wall, and treatment is marsupialization or excision.
Type IV cysts are multiple and involve both intra- and extra- hepatic ducts. Type V cysts are intrahepatic cysts and the rarest type. They are sometimes associated with congenital hepatic fibro- sis and medullary sponge kidney.
Treatment of type IV and V cysts depends on the extent of liver tissue involved. Excision, often requiring Roux-en-Y jejunostomy, is preferred over drainage. However, resection may not be an option at all, such as in the case of multifocal cysts involving both lobes with background hepatic fibrosis.
Which of the following statements is true regarding intrahepatic cholangiocarcinoma?
A. Survival following resection is generally lower than that for distal bile duct cancer.
B. Resection is contraindicated unless histologically negative margins can be obtained.
C. The best survival is achieved with liver transplantation.
D. Adjuvant chemotherapy improves survival following resection.
E. None of the above.
ANSWER: A
COMMENTS: Cholangiocarcinoma arises from the bile duct epithelium and can occur anywhere along the biliary tract. It constitutes 5%–20% of primary liver cancers.
Tumors arising from the extrahepatic bile ducts differ from those located intra- hepatically in terms of their clinical findings, therapy, and prog- nosis. Tumors of the extrahepatic bile ducts are typically manifested as biliary obstruction.
Intrahepatic tumors appear similar to hepatocellular cancer, a liver mass with absent or vague symptoms such as pain, weight loss, nausea, and anorexia. The treatment of choice is surgical excision, which is associated with a 15%–20% 5-year survival rate. The prognosis is best for tumors of the distal bile ducts that can be resected by a pancre- aticoduodenectomy.
Tumors involving the bifurcation of the bile duct (Klatskin tumor) are less often resectable.
Tumor size and the presence of satellite nodules are correlated with outcome.
Histologically negative margins are always desirable, but prolonged survival can be attained even with microscopically involved margins.
If the tumor cannot be resected, improved survival has been noted with bypass or stenting procedures. Liver transplantation for cholangiocarcinoma has been associated with frequent recurrence and has not generally been encouraging.
Adjuvant chemotherapy has not typically been useful for bile duct cancer.
A 75-year-old man with hepatitis C cirrhosis presents for an annual checkup. He denies any new complaints, including jaundice, abdominal pain, ascites, gastrointestinal bleeding, or encephalopathy. Temperature is 99.1°F, pulse 85 beats/ min, blood pressure 109/78 mmHg, respiratory rate 18 breaths/min, and saturation 98% on room air. What studies should be ordered?
A. None B. Ultrasound C. α-Fetoprotein (AFP) level D. Esophagogastroduodenoscopy (EGD) E. CT scan
ANSWER: B
COMMENTS: Hepatocellular carcinoma (HCC) is among the most common cancers worldwide. Anyone with cirrhosis is at risk, and some etiologies of cirrhosis are independent risk factors separate from cirrhosis. For example, in hepatitis B infection, HCC can develop without underlying cirrhosis. Other populations at risk include patients with hepatitis C, alcoholic liver disease, hemochromatosis, alpha-1-antitrypsin deficiency, primary scle- rosing cholangitis, aflatoxins, HA, anabolic steroid use, and pesticides.
HCC screening is therefore recommended in all individuals with cirrhosis. The recommended screening method is ultrasonography every 6 months.
Ultrasonography is 60%–90% sensitive, depending on the size of the lesions. Adding AFP testing is no longer recommended due to its cost and false-positive rates.
A 69-year-old woman with Child’s C cirrhosis has an elevated AFP and a new liver mass on ultrasound. Triple- phase contrast CT confirms a 3.5 ×4-cm2 mass with arterial enhancement and venous washout. She is other- wise healthy with no cardiopulmonary disease. What is the next most appropriate step in workup and treatment?
A. Interventional radiology (IR) biopsy and pathology B. Neoadjuvant chemotherapy C. Liver resection D. Liver transplant E. Assess portal vein pressures
ANSWER: D
COMMENTS: In a patient with positive HCC screening, diagno- sis is generally made by imaging. Triple-phase magnetic resonance imaging (MRI) is more sensitive and specific than triple-phase CT, but the latter is more often used.
The characteristic description of HCC is a lesion with arterial enhancement and venous washout. Biopsy is usually not necessary, except for unusual cases in which the clinical diagnosis is still in doubt.
Management of HCC is resection, and the most important factor in assessing candidacy for liver resection is a hepatic reserve.
In general, a patient with Child’s A cirrhosis can tolerate up to 50% liver resection, and Child’s B up to 25% liver resection. Liver resection is contraindicated in Child’s C cirrhosis, and trans- plant has better survival than resection in these patients. Portal vein hypertension must also be taken into account, as liver resec- tion may not be tolerated if portal vein pressures are above 10 mmHg. The Barcelona Clinic Liver Cancer (BCLC) staging system is often used to guide HCC treatment based on patient and tumor factors.
Liver transplantation is indicated in Child’s C cirrhosis as long as the patient is thought to be able to tolerate this.
The Milan criteria have been proposed to define HCC tumors in Child’s C cirrhosis that would benefit from transplantation.
These are tumors in which there is no extrahepatic disease, no macrovascular invasion, and either one tumor up to 5.0 cm in size, or up to three tumors and each up to 3.0 cm in size.
Patients with a new HCC diagnosis are awarded additional model for end-stage liver disease (MELD) exception points to minimize waiting list dropout due to disease progression.
Appropriate use of locoregional therapies in patients with
HCC includes all of the following except:
A. An attempt at down-staging tumors initially regarded as
unresectable
B. To maintain a patient within transplant listing criteria
C. Treatment of multiple small tumors that are multifocal
D. Treatment of tumors that are in locations of anatomic constraint
E. An alternative to resection in patients with good hepatic reserve
ANSWER: E
COMMENTS: Locoregional therapies, such as radiofrequency ablation, transarterial chemoembolization (TACE), and radiation, have been used with the aim of down-staging tumors initially regarded as unresectable. The resectability of a tumor is determined by a number of factors including hepatic reserve, patient functional status, and tumor size. Resection is still preferred over locoregional therapy and should be performed in patients without contraindication.
Additionally, locoregional therapies have been used in patients with cirrhosis who do not qualify for liver transplantation based on the Milan criteria, in efforts to bring or maintain them within transplant listing criteria. Other uses include treatment of multiple bilobar tumors unresectable due to distribution and tumors in loca- tions that are difficult to access surgically. Finally, another role for these modalities is in the treatment of liver metastases from colorectal cancer.
A 55-year-old woman with a history of colorectal cancer 10 years ago, treated with right hemicolectomy and chemotherapy and with no evidence of disease since then, presents with vague abdominal pain. Temperature is 98.7°F, pulse 67 beats/ min, blood pressure 132/78 mmHg, and saturation 99% on room air. Labs are remarkable for AST 67 IU/L, ALT 83 IU/L, and carcinoembryonic antigen (CEA)-27 ng/mL. Triple-phase contrast CT shows a new peripheral liver mass that slightly enhances on arterial phase. She has hypertension that is well controlled with medications. What are the most likely diagnosis and most appropriate management of her liver lesion?
A. FNH; observation
B. Primary liver cancer; resection
C. Primary liver cancer; chemotherapy
D. Metastatic colorectal cancer; resection
E. Metastatic colorectal cancer; palliation
ANSWER: D
COMMENTS: Metastatic liver tumors are much more common than primary liver tumors. The liver is the most common site of colorectal cancer metastasis, followed by the lung. Other common sources of liver metastases include pancreas, lung, and breast carcinomas. A new hepatic lesion with a known history of one of these carcinomas is generally sufficient for diagnosis, without a need for biopsy. CEA is usually elevated as well if colorectal cancer is the primary.
Characteristics that further support metastasis over primary liver cancer include a history of primary cancer, peripheral residing lesions, multiple lesions, and mass hypovascularity (hence only slightly enhancing compared with adjacent liver parenchyma).
Resection of hepatic metastases from colorectal cancer pro- vides a clear survival advantage over any other treatment and should be performed whenever possible. Over the past two decades, improvements in intraoperative techniques have afforded improved outcomes in liver surgery. Experienced centers demon- strate 5-year survival rates of 25%–40% with mortality rates of less than 5%.
Candidacy for resection is similar to that in HCC and, in other words, depends highly on hepatic reserve. In general, an isolated liver and/or lung metastasis in a patient with colorectal carcinoma should be resected unless contraindicated.
This patient should also undergo surveillance colonoscopy to evaluate for local recurrence.
Which of the following is the most accurate method for
identifying hepatic metastases?
A. Transabdominal ultrasound B. CT C. Laparoscopy D. Intraoperative palpation E. Intraoperative ultrasound imaging
ANSWER: E
COMMENTS: Transabdominal ultrasound is as accurate as CT for detecting liver tumors that are 2 cm in size or larger. For smaller lesions, CT is more accurate, although it can miss the smallest lesions (<1 cm). Laparoscopy is useful for identifying small metastases on the liver or peritoneal surfaces that escape discovery by noninvasive preoperative imaging modalities.
Laparoscopy has been incorporated into the staging workup of a variety of intraabdominal malignancies, including those of the liver. However, one of its limitations is its ability to assess the interior structure of solid organs. It is now well recognized that intraoperative ultrasound is the most accurate method for detecting and assessing hepatic tumors. Not only does intraoperative ultrasound discover more lesions than any other modality (including palpation), but it also clearly demonstrates the anatomic relationship of tumors to important vascular structures, which is a critical determinant of resect- ability and the extent of resection necessary.
Intraoperative ultrasound can be performed with handheld or laparoscopic trans- ducers. Experience with intraoperative ultrasound for liver tumors has shown that the sonographic findings affect the surgical manage- ment of one-third to one-half of patients. Intraoperative ultrasound imaging has become an indispensable component of hepatic surgery.
A 56-year-old woman with hepatitis C cirrhosis has had a worsening mental status that has now progressed to hepatic coma. Which of the following can be used for initial treatment of a patient in a hepatic coma?
A. Reduction of dietary protein to 50 g/day or less B. Control of active bleeding C. Lactulose D. Neomycin E. All of the above
ANSWER: E
COMMENTS: Treatment of hepatic encephalopathy and coma is aimed at limiting the nitrogen that the liver must metabolize by eliminating nitrogenous material from the gastrointestinal tract and by inhibiting its absorption. At the same time, precipitating causes are sought and treated.
Nutritional support is important and can be initiated with standard amino acids and restriction of dietary protein.
Cessation of any gastrointestinal bleeding from varices is an important step in reducing the conversion of intraluminal blood to ammonia.
Lactulose acts as a cathartic and also inhibits the absorption of ammonia by acidifying the colon.
Nonabsorbable antibiotics, such as neomycin and kanamycin, reduce colonic flora and the production of ammonia.
Systemic antibiotics may be useful for treating specific infections that precipitate encephalopathy but are not indicated empirically. Because the colon is the major site of ammonia absorption, colon resection or exclusion has been suggested to improve encephalopathy but is not a widely used therapeutic measure.
A 43-year-old man with alcoholic cirrhosis has had increas- ing abdominal distention over the last month. His vital signs are stable, and he is afebrile. Physical examination reveals a distended abdomen with a fluid wave. The initial management of the patient’s ascites should include all of the following except:
A. Transjugular intrahepatic portosystemic shunt (TIPS) B. Sodium restriction C. Diuretic administration D. Fluid restriction E. Diagnostic paracentesis
ANSWER: A
COMMENTS: Ascites is the most common major complication of hepatic cirrhosis. It is associated with a 2-year survival rate of 50%, and its onset in a cirrhotic patient should prompt an evaluation for liver transplantation.
Treatment of ascites depends on its cause, and therefore diagnostic paracentesis is required after a history and physical examination.
Abdominal ultrasound can confirm the presence of ascites if it is not certain by examination.
The serum-ascites albumin gradient is useful diagnostically. A high gradient (1.1 g/dL) indicates portal hypertension and suggests that the patient will be responsive to medical management consisting of sodium restriction (2000 mg/day) and oral diuretics.
Usually, both spironolactone and furosemide are administered to produce fluid loss and natriuresis. Spironolactone alone may cause hyper- kalemia, and furosemide alone is less effective.
Medical therapy controls ascites in about 90% of patients. When the ascites is refractory, serial therapeutic paracenteses (with or without the administration of albumin or other plasma volume expanders) are indicated. Liver transplantation is the ultimate treatment.
A peritoneovenous shunt is an option for patients with refractory ascites who are not transplantation candidates or who cannot undergo repeated paracenteses.
These shunts are fraught with potential complications, however, and do not prolong survival in comparison with medical management.
TIPSs or operative side-to-side–type portosystemic shunts may control the ascites in select patients.
What is the preferred site for needle entry in paracentesis?
A. 3 cm medial and 3 cm superior to the anterior superior iliac supine (ASIS) on either side
B. 3 cm medial and 3 cm superior to the ASIS in the right lower quadrant (RLQ)
C. 3 cm medial and 3 cm superior to the ASIS in the left lower quadrant (LLQ)
D. 3 cm medial to and at the level of the ASIS on either side E. Midline at linea alba, inferior to arcuate line
ANSWER: C
COMMENTS: The mechanism of ascites in cirrhosis is thought to be a result of the local release of vasodilators, such as nitric oxide. This causes splanchnic arterial vasodilation, leading to a reduction in effective arterial blood volume. The impending result is systemic vasoconstriction and renal sodium-retention, leading to total body fluid retention.
The serum-ascites albumin gradient (SAAG) is used to help determine the etiology of ascites. SAAG-serum albumin – ascitic albumin. SAAG ≥ 1.1 is consistent with portal hypertension, such as from cirrhosis, Budd-Chiari syndrome, portal vein thrombosis, congestive heart failure, liver lesion, and alcoholic hepatitis. SAAG < 1.1 points to other nonhepatic etiologies, such as peri- toneal carcinomatosis, nephrotic syndrome, and pancreatitis. Diagnostic paracentesis should be performed on all patients with new-onset ascites, as the etiology of the ascites will guide treatment.
The management of ascites due to portal hypertension includes dietary sodium restriction, fluid restriction, and diuretics. For ascites refractory to medical therapy, treatment options include large- volume paracentesis, TIPS, and peritoneovenous shunt. Liver trans- plantation is the definitive treatment, and ascites in a patient with cirrhosis should prompt an evaluation for liver transplantation.
In paracentesis, the generally accepted site for needle entry is 3 cm medial and 3 cm superior to the ASIS in the LLQ. Although controversial, a common practice is to follow paracentesis with albumin 1-g infusion for every 100 cc removed, in an attempt to maintain plasma oncotic pressure.
A 43-year-old woman with primary biliary cirrhosis compli- cated by massive ascites, hepatic encephalopathy, and multiple episodes of upper gastrointestinal bleed requiring endoscopic management presents with hematemesis. Temperature is 98.7°F, pulse 109 beats/min, blood pressure 103/77 mmHg, respiratory rate 20 breaths/min, and saturation 97% on 2 L of nasal cannula oxygen. After acute resuscitation with intravenous fluids and blood products, EGD is per- formed showing multiple bleeding esophageal varices, which undergo endoscopic band ligation. When the patient is stabilized, TIPS is considered. Further workup including chest radiography shows a large right pleural effusion. What is the most important relative contraindication for portovenous shunting?
A. Hepatic encephalopathy B. Concern for continued active hemorrhage C. Massive ascites D. Pleural effusion E. No contraindications to TIPS
ANSWER: A
COMMENTS: Portal hypertension is responsible for the majority of the morbidity and mortality associated with cirrhosis, such as variceal bleeding, refractory ascites, and hepatic hydrothorax. First- line therapy for patients with primary variceal bleeding is endo- scopic therapy with variceal band ligation or sclerotherapy. However, there is a high risk for rebleeding not amenable to endo- scopic techniques (refractory bleeding) or continuation of bleeding (recurrent bleeding).
This patient has failed medical and endoscopic management of variceal bleeding. TIPS is very effective at reducing or normal- izing portal pressure, the underlying cause of variceal bleeding. Other indications for TIPS include refractory ascites, hepatic hydrothorax, and Budd-Chiari syndrome refractory to anticoagulation.
The mechanism is via diverting blood into systemic circulation and decompressing the portal system. Because hepatic metabolism is bypassed, hyperammonemia may occur manifesting clinically as worsening hepatic encephalopathy.
Hepatic encephalopathy is therefore a relative contraindication to TIPS.